You are on page 1of 63

FEDERAL TAX OUTLINE

FALL 2002

PROF ZELINSKY

INCOME
61 Commissioner v. Glenshaw Glass (pg 90) Are punitive damages income? Glenshaw Glass was awarded punitive damages but reported only non-punitive damages Holding: Taxpayer looses Punitive damages are income and taxable Test: o Accession to wealth o Clearly realized o TP has complete dominion or control over

CASH -- NOT INCOME Cash isnt included in 61 LOANS NOT INCOME A loan is the most common form of cash dont have to pay income taxes on loans bc there is an offsetting obligation to pay There is no income when you borrow money ILLEGAL INCOME 61 doesnt say legal income only. Gilbert v. Commissioner (pg 220) Gilbert illegally took money but left a promissory note His intent was to pay it back. He pleads guilty to embezzlement charges for taking the money. Holding: Not taxable income bc it was a loan If it was really stolen, it wouldnt be a loan, so it would be income

BARTER INCOME Barter is taxable income to both parties. When you barter products or services you are taxed on the FMV of what you receive. Revenue Ruling 79-24 (pg 88) IRS says both housepainter and lawyer who trade services are income Imputed Income (self-performed services) are not income

DEPOSITS v. ADVANCE PAYMENTS Commissoner v. Indianapolis Power Requires customers w/ bad credit to make deposits with it to assure prompt payment of future bills. Customers can get a refund or apply amount towards bills if they pass a credit check. Deposits are under the companys complete use and control. Supreme Court says its NOT INCOME o Compares it to a loan o Company doesnt have complete control at time payments are made o Not advance payments

EXCEPTIONS TO 61
102: GIFTS & INHERITANCE (a) General Rule: NOT INCOME (c) Employer to Employee gifts: INCOME (gifts given in the business relshp are taxable)

Commissoner v. Duberstein (pg 95) Gave business associate a Cadillac as a gift Holding: Tp loses Not a gift so INCOME TEST: Gift is not taxable if: o Given w/ detached and disinterested generosity o Look to donors intent

Stanton (pg 101) Retiring priest was given money from ER when he resigned. Held: NOT INCOME

Congress responded to Duberstein w/ Section 102(c)(1) Overrules Stanton US v. Harris (pg 105) Man gae gifts to 2 prostitutes and they didnt pay tax on the money Held: Gift so NOT INCOME Based on Duberstein test of intent of donor. Prof. says this is wrong: Income test purpose is to determine capacity to pay The women certainly have capacity to pay

Olk v. US Craps dealer receives tips and did not report them as income Held: Tokes are taxable Not gifts Not detached and disinterested generosity

Pg 118 Questions 1. (a) Is this a gift, and if so how is it different from Olk? (b) 2. Is her $ given disinterested and detached?

74 -- PRIZES AND AWARDS Prizes and gifts are taxable. -- INCOME Prior to 1986, prizes and gifts were non-taxable and followed the rest of 102 Exception under 74(b) -- not taxable if given for charity (religious, charitable, scientific educational, artistic, literary, or civic achievement. Must be 3 Criteria: o Recipient selected w/o any action on his part to enter the contest o Recipient not required to render substantial future service as a condition, and o Prize is transferred by the payer to gov. unit or organization described in 170(c)

117 -- SCHOLARSHIPS AND FELLOWSHIPS General Rule: NOT INCOME Criteria for tax-free: o Must be a qualified scholarship (tuition, books and related services not room and board) o By a degree candidate o If you have to provide services for the scholarship, it is not tax free Exceptions: 117(c) o Making a person have to teach a course in order to get the scholarship is considered wages so its taxable o Half of scholarship is dependable on teaching Half the money is taxable, other half is not

o o

Athletic scholarships (free tuition to play football)? Maybe taxable under the quote other services? But, to date, college athletes are not taxed Could argue that it is income Merit scholarship (that require a certain GPA)? Is maintaining a certain GPA an other service?

85 -- UNEMPLOYMENT Congress changed to make unemployment benefits INCOME soTAXABLE Changed from pre-1986

86 -- SOCIAL SECURITY PARTIALLY TAXABLE AS INCOME Gross Income includes SS if adjusted gross income + 1/2SS > Base Amount Base amount equals $25,000 or $32,000 if a joint return (see 86(c)(1)) Taxable income equals the lesser of: o SS benefits received during the year or o excess under 86(b)(1) Most people dont pay income tax on SS A married couple can have an income of $32,000 before they get taxed on SS Zelinsky says 86 arises out of the politics of the AARP

E.g. Suppose you get $18,000 in SS. 0 + $9,000 < $25,000 -------- $0 is taxable. If you have $21,000 income from other sources. $21,000 + $9,000 > $25,000 ------- Reports $5,000 as taxable income. McAdams v. IRS 86(c)(1) W/o 86(c)(1)(c) Couples would always file separately to each get the benefit of the $25,000 base amount o Congress put in (c)(i) to prevent couples from filing separately Argument to support courts decision is that need bright line rules to make it clear when people should file together. Language of statute clearly says does not live apart from his spouse at all times during the taxable year

101 -- LIFE INSURANCE General Rule: NOT INCOME SO NOT TAXABLE One way is to say this is an extension of 102 and life insurance to a beneficiary is like a gift, so its right not to be taxable Any interest payments earned on the life insurance amount is taxable

79 -- GROUP TERM LIFE INSURANCE General Rule: Premium paid for the life insurance is taxable to the EE to the extent that such cost exceeds the sum of: (1) The cost of $50,000 of such insurance, and (2) The amount (if any) paid by the EE toward the purchase of the insurance Premium ER pays is excludable from gross income for the first $50K of insurance You pay the tax on the cost of such insurance, but the determinate is based on the life insurance coverage amount:

$50,000 insurance for $200 - $0 taxable income $75,000 for $300 - $100 taxable income US v. Drescher (pg 313) ER bought an annuity contract for $5,000. IRS argues that Drescher has income at the time the policy was purchased by the ER. Drescher argues that it is income when he gets the payout Holding: Tp loses. Premium paid by ER is INCOME to EE so its TAXABLE NOW bc EE is receiving an economic benefit now by knowing he has such an annuity. Arguments: o Wrong No accession to wealth bc it isnt transferable EE doesnt own the policy o Maybe the courts idea of economic benefit is wrong Test: Whether taxpayer receives an economic benefit How to value the benefit? How would EE pay if he isnt yet collecting $? Dissent: Doesnt matter that the policy isnt assignable it is still taxable at the full amount

FRINGE BENEFITS 119 -- ER provided provisions for MEALS & LODGING

Benaglia v. Commissioner (pg 53) Hotel manager received room and board as compensation. Held: NOT INCOME. Test: Whether benefits were provided strictly for the convenience of the employer. Zelinsky says this is a ridiculous decision agree w/ dissent

Congress responded to Bengalia w/ 119 Codified Bengalia 119 General Rule: Value of meals or lodging furnished to EE, his spouse and dependents by the ER for the convenience of the ER are NOT INCOME Criteria for excludable income o For the convenience of the ER (ratifies Benaglia) o Only if the meals are furnished on the ERs business premises o For lodging, EE must be required to accept such lodging on the business premises of his ER as a condition of his employment Issue: How to interpret condition of employment?

107 -- RENTAL VALUE OF PARSONAGES (house for priests) NOT INCOME Gross Income does not include: o Rental value of a home furnished to him as part of his compensation o Rental allowance to him (cash to rent a house) This section differs from 119: o Doesnt have to be at convenience of the ER o Not on business premises o Can give money to clergymen and it is excludable (119 is only for meals & lodging)

1984 Treasury convinced Congress to take a comprehensive approach to fringe benefits - 132 132 -- CERTAIN FRINGE BENEFITS GENERALLY FRINGE BENEFITS ARE TAXABLE UNDER 61 7 EXCLUDABLE FRINGE BENEFITS (not income so not taxable): 132(a) (1) No additional cost services (2) Qualified employee discounts (3) Working condition fringe (4) De minimus fringe (5) Qualified transportation fringe (6) Qualified moving expense reimbursement (7) Qualified retirement planning services 132(j) No discrimination in giving these benefits 132(b) No additional Cost services defined: o EE can exclude any service provided by the ER if: The service is offered for sale to ordinary customers in the ordinary course of the ERs line of business to which the EE works, and The ER incurs no substantial additional cost (including foregone revenue) o NOTE: Only applies to ER-EE relationship (not independent contractors, like consultants) 132(c): Qualified EE Discount Defined o E.g. Department store EE gets discount o Employee does not have to include this fringe benefit as GI as long as discount doesnt exceed (a) If property, gross profit percentage, (sales price cost) or (b) If services, 20% off price offered to customers o What this means is that an employee can purchase a good (from his employer) at least at cost price (but not for less or that will be considered income to employee) or if its a service he can only get a maximum of 20% off. (If its more then 20% then the whole amount {not only the % over 20%} is considered as income)- to bad! o Note: this is only allowed for the goods sold or services within the employers line of buz in which the employee performs services that are available to non employee customers. - The form of such a qualified fringe can be reduction or a cash rebate. 132(e) De Minimus Fringe o is there a theory of administerability or a political maneuver. Use of the gym: 132(j)(4) o Must be on-premises to be tax-free benefit o On-premises can company rent the space for the gym or do they have to own? Qualified Retirement Planning Services 132(m) o ER pays for EE to get tax advice: 132(m) is this income?

These rules are deeply troubling bc the EE is getting an economic benefit that makes him better off that another Ee who isnt getting the same fringe benefits. 117(d) -- QUALIFIED TUITION REIMBURSEMENT General Rule: Any qualified tuition reduction plan is NOT INCOME (ER paying tuition for EE is generally NOT TAXABLE to EE) Has the discrimination element

125 -- CAFETERIA PLANS Ees pick tax-free fringe benefit or income General Rule: NOT INCOME just bc there is a choice

104 -- COMPENSATION FOR INJURY AND SICKNESS General Rule: NOT INCOME The following are excludable from income: o Amts received under workers compensation This is wage replacement, so why isnt it taxable if wages are? Is this right? Cant be bc a pitcher for the Yankees on the disabled list get tax free workers comp o Amt received in damages for injuries hypo: A is in a car accident, sues, and wins $5 in compensatory damages and $3 in punitive damages $5 is not taxable, but $3 is taxable Hypo: No physical injury, but are mentally a wreck Taxed on all damages received Amounts received for emotional damages are income and taxable. o 104(a)(3): Amounts received through accident or health insurance for personal injuries or sickness if the EE paid the premiums on the insurance. If the ER pays the insurance premiums, then the portion of the proceeds attributable to the ERs contribution is taxable unless the insurance proceeds reimburse the taxpayer for extensive medical care w/ 213. People who buy their own medical insurance and receive amounts through the plan, the amount he collects is tax-free

105 AMOUNTS RECEIVED UNDER ACCIDENT & HEALTH PLANS General Rule: TAXABLE. Amounts Ees get through a health insurance from an ER-paid plan are generally taxed on the amount 105(a) Exceptions: 105(b) o Income does not include amounts paid directly or indirectly to the tp to reimburse the EE for expense incurred by him for the medical care o Hypo: Dean pays professors premium (tax-free to EE), EE gets money from the plan when he gets sick (tax-free????)

104(a)(3) v. 105: People who buy their own medical care and those whose ER pays are treated differently

106(a) CONTRIBUTIONS BY ER TO ACCIDENT & HEALTH PLANS General Rule: Gross income does not include ER-provided coverage under an accident or health plan If ER paid the premium then that is not considered income for the EE, but the benefits of the policy will be income (taxable) to the EE (105) Zelinsky says this is a big mistake : If one employee makes $30K/year w/o medical insurance and another makes $30K/year and medical insurance doesnt that second person have a greater accession to wealth

103 INTEREST ON STATE & LOCAL BONDS Generally, under 61, interest is income o Interest from bonds from a corporation are income But, under 103, interest from state or local bonds are NOT INCOME Elaborate limitations to restrict the use of these bonds A bond is an IOU lending money to the government (b) EXCEPTIONS: The interest on the following is income and therefore, taxable: o Private Activity Bonds which isnt qualified (141) Interest from private bonds are taxable unless the private bond is qualified under 141 Specific exemptions for private activity bonds which would make them qualified are bonds that include airports, docks, mass commuting facilities, hazardous waste facilities, certain gas & electric enterprise zone facility bonds (141) o Arbitrage Bonds (148) o Bond not in registered form (149)

South Carolina v. Baker Interest on municipal bonds are not tax-exempt unless the bond is registered. Held: The court changed its mind and said interest can be taxed So, why do we still have tax-free municipal bonds in 103? o States get to pay a lower interest rate and higher bracket tax payers get away tax free

141 PRIVATE ACTIVITY BONDS; QUALIFIED BONDS General Rule: Private Activity Bonds are NOT EXEMPT Private Buz tests: (how do I know if its a private bond?) 141(b) 1) If more then 10% of proceeds of issue are used for any private buz use. 2) Private security/pymt test: if pymt of principle of (or interest on) +10% is directly or indirectly a) secured by any interest in property used for private buz use or pymts for property, or b) derived from payments in respect of property or borrowed money used or to be used for a private buz purpose. Private loan financing test `141(c) o If amounts of proceedsexceeds lesser of 5% of proceeds or $5 million----then it is a private bond and is taxable. Qualified bonds: how can I make a private bond qualified? 1) It can become qualified and there4 interest will not be income if the private activity bond is for private activity bonds for Exempt facility bonds (142), or qualified mortgage bonds (143) or small issue bonds (144) or a qualified student loan or a qualified redevelopment bond or a qualified charitable bond. AND 2) Meets volume cap on issuance ($50 of private activity bonds per resident per year or state population ceiling (146d) AND 3) IT MUST BE A REGISTERED BOND FOR IT TO BE QUALIFIED. Pt is: the interest on a private activity bond is not income (not taxable) if o Its a qualified bond under 141(e)(1) AND o It meets volume cap (to be qualified u must meet this volume cap)146(d). o And is registered.

108 DISCHARGE OF INDEBTEDNESS INCOME General Rule: TAXABLE INCOME 61(a)(12) EXCEPTIONS: 4 provisions where there is not income for discharge from indebtedness o If taxpayer is in a bankruptcy case (title XI) o If taxpayer is insolvent (Reflect policy that we would like you to pay tax, but realize you should get on w/ your life o Indebtedness discharged is qualified farm indebtedness farms or real estate (special interest legislation) apply if taxpayer is solvent or insolvent o If the indebtedness was a qualified real property business indebtedness (S corp., partnership, or LLC) 108 is a postponement bc the taxpayer pays in the future (108(b)) o If discharge of indebtedness is excluded from income then tax attributes of the taxpayer must be reduced So its not a full release, just a postponement 108(d): Definition of indebtedness o Defines indebtness of taxpayer, but the definition only applies to this section. Discharge of indebtness under 61 is not defined in the IRC. 108(f)(2): STUDENT LOANS o General Rule: Discharge from student loan is NOT INCOME if student worked for a certain time in certain professions. o Exception for student loans that are forgiven for certain programs

Kirby Lumber (pg 180) Kirby Lumber issues bonds (to borrow money) worth $1M and then paid back just $850K (bought back less than he owed). Held: Amount he didnt pay back was INCOME so TAXABLE

Zarin v. Commissioner (pg 184) Casino gives Zarin $3.4 million in chips and he gambles it all away. Casino goes to court and asks for it all back. The parties settle for $500,000. IRS says the difference is taxable income Held: NOT INCOME bc not a discharge from indebtedness. Zelinsky says this is discharge of indebtness so the court was WRONG this is the same case a Kirby Lumber. Doesnt matter that there are restrictions on how the money can be used most loans have restrictions (e.g. mortgages must be used for the house) Other argument is the Zarin never touched $3.4 million - he just had chips. o Unlike Kirby, Zarin could only use the chips w/in the casino so he never really had $. o Kirby had $ and had to give the IRS part of it, but Zarin has no $ in his pocket, so should he have to give the IRS a part of it?

Centennial Savings Bank (borrower) imposed a penalty if a CD owner (lender) withdrew funds early so bank had to pay less back to the CD owner. Bank argues that the amt received from the discharge of indebtedness is not income argues its qualified business indebtedness 108(a)(1)(C). Held: Surcharges were INCOME to the bank taxable relief provisions dont apply

Diedrich v. Commissioner (pg 194) Facts: Parents give 3 children a gift of stock on the condition that the kids pay the gift tax. Issue: Was the parents relief of paying the gift tax income? Holding: Yes, DONOR HAS INCOME from discharge of requirement to pay gift tax. Parents Basis = $51,000 Becomes kids basis (1015) Gift tax = $63,000 Income = ~ $11,000

72 ANNUITIES (PENSIONS) 72 is important bc arguably it could simplify the Code formula could be applied to other sections (life insurance and SS) Annuity a contract w/ an insurance company; insurance company makes money by pooling premiums from the buyers and insurance company makes payment to the annuitant. General Rule: gross income includes annuity payments. Issue arises when buyer and annuitant and the same person Exclusion Ratio - 72(b) o A certain portion of each annuity payment is not taxable the Exclusion Ratio helps us figure out this amount Amount of investment in the contract / the expected return (using how many years you expect to get the annuity) Then you take this # and multiply by what you are getting each year This number is the amount of the annuity that is tax-free.

EXCLUSION RATIO = (INVESTMENT IN THE K/EXPECTED RETURN) * ANNUITY AMT PER YEAR (this equals the amount that is tax free per year)

72(c)(1) o The investment in the contract is the aggregate premium What if you live longer than expected? o 72(b)(2): If you live longer and you are still collecting the annuity, you may not exclude it Now the whole amt is taxable. The exclusion ratio can only be used to recover the investment o 72(b)(3): If you die before the time that was expected, the remaining amt of what you invested gets a deduction in the year that you die.

Examples: 1. TP pays premiums of $500. Retires at age 60. Has 20-year life expectancy. Gets $100/yr. Investment in the contract = $500 Expected return = $2,000 ($100 * 20) Exclusion Ratio = $500/$2,000 = If individual gets $100, $25 is tax-free, $75 is taxable. If individual lives the full 20 years, all payments become taxable after 21 years. 2. Individual age 65 is expected to live for 15 years, invests $100 initially and gets $50 (total of $750). Exclusion Ratio is 100/750. [these numbers might not be right check] 100/750 * $7.50 = $1.00 Taxpayer gets $1.00 tax-free and pays tax on the rest.

3. Suppose taxpayer gets $7.50 payment and then dies. What happens now? o Before 1986, too bad never get $100 back. o Today, taxpayers estate can deduct to the extent they didnt recover the $100 tax-free (72(b)(3)) 4. Suppose taxpayer lives a long time. At age 80, they get back all the $100 tax-free. Exclusion ratio wont apply after person turns 81. Which system makes more sense (pre-1986 or later)? Pre-1986 was much easier - no complexities. 1986

DIVORCE: ALIMONY, PROPERTY SETTLEMENT & CHILD SUPPORT

71 ALIMONY & CHILD SUPPORT General Rule: Alimony is INCOME 61(a)(8) Alimony Defined 71(b) Basic Rules o Must be in cash o Must be received under a written divorce or separation document o The divorce documents cant designate the payment as payment o Spouses cant live in the same household o There is no obligation to continue payments after the payee spouse dies 71(c) Child support is not included as income o 71(c) does not mention that child support is only for minors Litigable Issues: o Wife (payee) wants to call it child support; husband (payor) wants to call it alimony!!!!!

215 ALIMONY DEDUCTIONS Payor of alimony gets a deduction 215(b) no deduction for child support

1041 TRANSFER OF PROPERTY BTW SPOUSES OR INCIDENT TO DIVORCE General Rule: No gain or loss recognized on the transfer of property btw spouses or btw former spouses if incident to a divorce CARRYOVER BASIS: 141(b) Transferee get transferors basis

US v. Davis (pg 361) As part of a divorce settlement, H gave W 1000 shares of stock. Hs basis was $1.00 per share and at time of transfer they are worth $5.00/share. Held: H had to pay tax on the gain. Is this right? o 1001 gain from the sale or disposition of property is taxable There has been a disposition o This is not a gift not detached or disinterested o Court is wrong on one important issue Mrs. Davis also has income bc she has disposed of her rights and got stock Mr. Davis has income bc he was released from liability (discharge of debt) Maybe it is right that both have income bc they exhange (exchanges of property/barter are taxable). o Zelinsky says court was wrong bc of the exchange barter is taxable! (barter of cash is not taxable)

Congress overruled Davis w/ 1041 No gain or loss for transfer of property bc spouses or former spouses bc of divorce 1014(b) o Treat property like a gift o Classic carry over basis The gain will be realized when W eventually sells the property

Problems pg 366 1. $200,000 to Wilma is not alimony (not cash), not child support, so its property. She does not have income under 1041 bc transfer of property is not a taxable event. (b) W/o the note, Henrys basis would be $100,000 under 1041 (carry-over basis, like a gift). But, with the promissory note, he does not have basis in the note bc its property in the form of cash. 2. Gain = $300,000

Interpretive Issues under 1041 Question of a stock redemption

Craven v. US Pursuant to divorce settlement, Linda agrees to sell her stock back to the company and gets a promissory note of $4.8M back (money). Her husband guaranteed the note. Linda did not report the imputed interest income on the note or capital gains from the redemption of her stock. Held: Transfer to the corporation fit under 1041 so there is no recognition. Court says the husband should pay tax, not the wife Zelinsky says this is wrong bc the wife made money so she should pay tax o Linda has no basis and now she has money, so she made a profit

10

o o

Holding is wrong bc she sells stock and makes money Under 61 and 1001, this is income. 1041 should not apply it was not a transfer from wife to the husband Wife transferred to the corporation, not to her former spouse. Both parties benefited from this transaction Wife sold to who had the money and husband got rid of her as a minority shareholder Under 1001(b), the test is who received the money Wife did, so she should pay tax

11

RECOGNITION RULES
RECOGNITION OF GAIN OR LOSS -- 1001 Fluctuations in value are not recognized as income Rule of Realization: a sale or disposition of property must occur to realize income Hypo: Buy cup for $1 and find out it is worth $200 No income until cup is sold o If you sell the cup for $200, you pay tax on $199. 1001(a) Computing Gain or Loss o GAIN = Amount Realized/Adjusted Basis = Sale Price - Cost 1001(b) - Amount Realized o Money received + FMV received o Cup is worth $200, B trades cup for a $200 tie. Barter is taxable $200 is amount realizes whether cash or property in-kind o Basis=sale price

3 TYPES OF BASIS 1) 1012 basis of property cost basis The basis of property shall be the cost of such property.

2) 1014 Basis of property acquired from a decedent step up basis (A)(1) When a person gets property from a decedent, the donees basis is the FMV of the property at the time of decedents death. Ex: I buy tie for 10 and turns out its worth 50 then I leave it in my will to Joe- and then Joe sells it for 70. What is the adjusted basis Joe should use? Joe should use the amount the tie was worth when I diedand that was 50. So when Joe sells it for 70 we calculate 70- 50= 20. 20 is the amount earned that is taxable. 3) 1015 Basis of property acquired by gifts and transfers in trust- carry over basis When donor gives property as a gift, the donee has the same basis as the donor- this is called the carry over basis. See Exception below for losses at time gift is made.

BASIS OF PROPERTY COST 1012 Basis = cost (defines basis as cost) E.g. A gives cup to B. o No income to B bc its a gift o When B sells it, Bs cost = ? (see 1015)

BASIS OF GIFTS -- 1015 Basis = Donors cost CARRY-OVER BASIS: Basis is the same as it was in the hands of the donor EXCEPTION: If the basis is greater than the fair market value of the property the basis to determine loss will be the fair market value. 1015(a) exception only applies when there is a loss at time gift is made. Basis means cost if you are the buyer, but means donors cost if you are the recipient E.g. B keeps cup and leaves it in his will to his daughter, C. C sells it. Cs basis is fair market value at time of Bs death.

BASIS OF PROPERTY ACQUIRED FROM A DECENDENT -- 1014 Basis = Fair market value at time of deceased death (here, basis does not mean cost) You get a step-up basis if you get something in a inheritance Inheritance gets rid of the tax on property Result is forgiveness of gain

12

Taft v. Bowers (pg 119) Father gave daughter stock. Fathers basis was $1,000. When father gave it to her it was worth $2,000. She then sells it for $5,000. B argues she should pay tax on $3,000, which is the appreciation that occurred in her hands. Held: She has gain of $4,000 bc her fathers basis is used. o Under the statute, Bs amount realized is the total purchase price of $5,000. Bs basis is $1,000 (donors cost - 1015). Bs gain is $4,000 Note: If B inherited the process, her basis would be $2,000

Problem 3 Pg 127 (a) Give her the stock and she has a gain, but its ok bc she is a low bracket taxpayer (b) Option 1: He sells it and he has a loss of $40,000 this is good bc he is a high bracket taxpayer; Option 2: He gives the stock to the daughter and she sells it at a basis of $80,000. Father should sell and give the daughter cash (c) If he sells the stock he has a gain and must pay tax. The alternative is to give daughter the stock. 3rd alternative is to give daughter the cash and he owes no tax on the stock. He should hold onto the stock bc then there is no tax to pay bc it is unrealized (d) Should sell the high basis stock he has a loss of $40,000. Option 2: If he sells the other stock, he has to pay the tax on the sale. Option 3: If he gave her the lower basis tax she pays tax on the sale. Option 4: Give her the higher basis tax and her basis is $80,000 (e) He should hold onto stock until he dies bc then the daughter gets a step-up basis under 1014

What happens if you die w/ a loss? Assume facts in 3(d) Under 1014, If father dies and fair market value is $80,000, daughter pays tax on $80K. Loss died w/ father. He should have sold before he died.

13

DEDUCTIONS
262 PERSONAL LIVING & FAMILY EXPENSES General Rule: No deductions for personal, living or family expenses

BUSINESS EXPENSES
162 TRADE OR BUSINESS EXPENSES 162(a) General Rule: Deductions for all ordinary and necessary expenses paid or incurred in carrying on any trade or business o Take deductions for the expense of earning income o Theses deductible expenses include: Reasonable allowance for salaries or other compensation for personal services Traveling expenses while away from home in the pursuit of a trade or business Rentals or other payments for continued use or possession of propertyREPAIRS Repair merely serving to keep property in an operating condition for the purpose for which it was used is an ordinary expense, thus deductible Like fixing a broken window

162 v. 262: Litigable issues over whether an expense is personal (262) or for trade or business (162).

Welch v. Helvering (pg 587) Taxpayer pays his companys debts in order to obtain the goodwill of customers for his new business. TP wants to deduct as business expenses. Held: Expenses for goodwill and reputation are not deductible bc not ordinary business expenses Zelinsky says Cardozo was wrong There is no way that this cant be deductible bc its Welchs only way to make good with his creditors This is a form of advertising o The only way for Welch to be able to do business with these people again is to pay them back this was the only ordinary and necessary way to do this o If Welch had taken out an ad in the newspaper saying he was sorry for bankrupting his creditors, that expense might be deductible, so why cant this case be deductible. Maybe paying them back is a form of advertising that his business is legitimate. o The best evidence that this is ordinary and necessary is that he pays off the creditors he has to salvage his own name

Gilliam v. IRS (pg 593) Tp flew on plane to go lecture and had an attack and acted crazy. He wanted to deduct the expenses of paying damagers from the lawsuit against him under 162 as business expenses. Held: Not deductible bc not directly related to his business activities not ordinary bus. expense Under 162(a) o Ordinary: Is this like a car salesman paying a tort action? Why is this not an unexpected cost attached as an additional expense to his deductible business trip? Dont you usually get a deduction for accidents that happen in the course of business? Or is this not an ordinary business expense? Isnt it not a common accident for someone traveling for business? o Necessary, Paid, Incurred during a business

263 CAPITAL EXPENDITURES General Rule: Not deductible (capital expenditures do not change net worth, but rather a transfer of the form of the wealth) Capital expenses include o Permanent improvements o New buildings o Anything with long-term benefit is basically non-deductible Building or equipment is deductible over time, but some capital expenditures are NEVER DEDUCTIBLE LAND

263 v. 162: Question is what is deductible expenses and what is a capital expense.

14

Encyclopedia Britannica (pg 551) Publisher EB hired another company to produce a book. EB says their payments to the outside writing were deductible expenses under 162 as ordinary and necessary business expenses. IRS says these payments are non-deductible capital expenses Held: Book was a capital expenditure, so not deductible. o Long-term benefit.

Problem was that out of house expenditures were not deductible, but in-house were. 263A CAPITALIZATION & INCLUSION IN INVENTORY COSTS OF CERTAIN EXPENSES If a cost is producing a long-term asset, it has to be capitalized, whether it is produced in-house (internal salaries) or outside (hiring someone else) Purpose is to limit things that are deductible Some self-created items are non-deductible (Encyclopedia Britannica must be capitalized) Applies to real or tangible personal property If EB writes an encyclopedia, the salaries of those who worked on the book are not deductible Zelinsky says this is a terrible statute it is unclear what is deductible ANTI-TAX SHELTER LEGISLATION

INDOPCO v. IRS Corporate takeover expenses that lead to long-term benefit have to be capitalized Held: Legal and investment fees incident to takeover are capital assets and cant be deducted Is this right? o This doesnt seem to be a permanent asset o Long-term benefit test is not useful bc then everything would give a long-term benefit

Midland Empire Packing v. IRS (pg 563) Repair and maintenance expenses oil proof a basement Held: It was a REPAIR so Deductible bc it was an ordinary and necessary business expense Is this right? o Is this a capital expense bc it was a permanent improvement? If they didnt do this, they would have to get a new building, so this might be like a capital expense. Maybe the liner extends the life of the building from the time the meat inspector gave the company the choice. o Fixing a broken window is deductible

Norwest Corporation v. IRS (pg 571) Held:: Asbestos removal was not a repair so its a capital expense not deductible Part of a general plan to improve the building Maybe this is wrong be it was an unexpected hazard

Why arent both cases repairs? Then they would be deductible. Revenue Ruling 94-38 (pg 568) IRS says groundwater treatment is a capital expense while the soil remediation and ongoing groundwater treatment is deductible. This Ruling goes against Norwest Norwest said you have to either label the entire expenditure as capital or deductible, but the Revenue Ruling says you can separate the expenses

How do these 3 situations fit together? -- 263A put in self-made expenses as capital expenses. (A reaction to Posners concern in Encyclopedia Britannica)

15

195 START-UP EXPENDITURES Start-up costs are neither immediately deductible or capitalized -- This section is the first instance where we permit a deduction over time for capital expenditures General Rule: No deduction for start-up expenditures, Exception: Can be amortization at the election of the tp deductions over a 60 month period 195(c)(1) Definition of Start-up expenditures: (A) paid or incurred in connection with: (i) Investigating the creation or acquisition of an active trade or business (ii)Creating an active trade or business (iii)Any activity engaged in for profits and for the production of income before the day the active trade or business begins (B) amounts which, if paid or incurred in connection with the operation or an existing business or trade, would be deductible in the year paid or incurred. If you already have a bus. and you want to expand, these costs will not be start-up, rather will be regular business expenses that can be deducted now.

Revenue Ruling 99-23 To explain what start-up costs are o expenses under 162, they are deductible immediately; o If they are 263 capital expenditures, no deduction; o Third category is start-up expenditures Ruling says that when company comes to the point in time of knowing the specific business, those start-up costs are not start-up costs and cant be amortized (they are capitalized under 263)

DEDUCTIONS FOR ILLEGAL ACTIVITIES


162(c) ILLEGAL BRIBES, KICKBACKS AND OTHER PAYMENTS No Deduction for payments made to officials or Ees of any government if it is a bribe 162(e) DENIAL OF DEDUCTION FOR CERTAIN LOBBYING & POLITICAL EXPENDITURES No deductions for: o influencing legislation o participating in any political campaign on behalf of any candidate for public office o Any attempt to influence the general public on elections or legislative matters

162(f) FINES & PENALTIES No deductions for fine or similar penalty paid to a government for a violation of a law o Does not refer to restitution to an injured private party (Stephens case)

162(g) TREBLE DAMAGES UNDER ANTI-TRUST LAWS No deduction for treble damages for antitrust

280E EXPENDITURES IN CONNECTION WITH ILLEGAL SALE OF DRUGS No deductions for drug dealing expenses

16

DEDUCTIONS FOR LOSSES


165 - LOSSES 165 (a) General Rule: deductions for any loss sustained and not compensated by insurance o If a loss has been offset by insurance, there is no diminishment of capacity to pay 165(b): Amount of Deduction: Basis for determining loss deduction shall be the adjusted basis in 1011. 165(c): LIMITATION ON LOSSES Loss deduction limited to: o (1) Losses incurred in trade or business o (2) Losses from any transaction entered into for profit (even if not connected w/ trade or bus.) o (3) Losses of property not a trade or business for fire, storm, shipwreck or other casualty or theft (Casualty Loss Exception) This is surprising bc 262 says no deduction for personal expenses 165(h): TREATMENT OF CASUALTY GAINS & LOSSES o (1) Casualty loss will only be allowed when it exceeds $100 (damages must be over $100) o (2): Casualty loss is deductible only to the extent that it exceeds 10% of the adjusted gross income If personal casualty losses exceed personal casualty gains, such losses will be allowed only up to the amount that is the sum of: Personal casualty gains, plus Excess of losses that exceeds 10% of AGI NOTE: Losses from gambling are deductible only to the extent of the gains from gambling (165(d)) 165(h)(4)(E): only a loss if individual files a timely insurance claim

Dyer v. IRS (pg 408) (cat breaks vase) Cat breaks a vase worth a lot of money. Vase was not insured. TP claims this is an other casualty Holding: No deduction Today this case wouldnt have occurred bc now you can only deduct for amounts lost over $100 and if its more than 10% of AGI A standard method to interpret undefined terms like other casualty is to put it in the context of the other terms Court says when you do this it is not similar. o Could make the argument that putting it in the context of the other terms allows for the deduction

Blackman (pg 411) TP sets his home on fire Holding: No deduction bc TP was grossly negligent Arguments o Statute says fire losses are deductible, so this should be deductible o Other argument is that this behavior should not be rewarded and it is understood that this type of negligent behavior should not be deductible

Chamales v. IRS (OJs neighbor) TP buys house in OJs neighborhood and it decreased in value by 30-40%. Tp wants to take this as a loss deduction. Held: No deduction for loss in value to house bc its only a temporary fluctuation - Steadily operating cause is not deductible.

Stocks that decline in value can be taken as a loss deduction under 165(c)(2).

165 (d) WAGERING LOSSES Need this provision so that people cant claim gambling as their business and deduct all losses If you lose, no deduction If you win, can deduct up to amount won What does wagering mean?

17

Whitten v. Commissioner (pg 480) Contestant on Wheel of Fortune wanted to deduct cost of bringing his family to the show. Held: No deduction Tension btw 162 and 165 Why doesnt he try a deduction under 162? o This isnt really a business or trade expense Court says no deduction under 165 Maybe he is entitled to a deduction under 212(1) production of income o Could argue he didnt go on the show for the production of income, so not deductible

Stephens v. IRS (pg 604) TP stole money (embezzled $530,000 from the company) and then makes restitution by paying back the money plus interest and deducts this amount. TP claims deduction under 165 (deductions for losses) Held: Court says he is entitled to this deduction o Use 162(a) Deductions allowed for Ordinary - Y Necessary - Y Expenses - Y Paid Y Carrying on a trade or business - ??? Is embezzlement a trade or business? 280E supports the courts decision: Congress specifically made illegal drug money non-deductible and did not make all illegal money deductible. Since his inflows were taxed, should his outflows be deductible? o But, isnt this a personal living expense (got out of jail) under 262? o This isnt a loss, its a gain he got out of jail

Questions pg 608 Are any of these expenses deductible?

183 ACTIVITIES NOT ENGAGED IN FOR PROFIT (HOBBIES) General Rule: No deduction if an activity isnt engaged in for profit (i.e. things not falling under 162)

Nickerson v. IRS (pg 460) Nickerson had a farm. He wanted a deduction under 162 as a trade or business expense. IRS said profit wasnt his primary goal it was a hobby. Held: Business expense so deductible under 162 Dont all people w/ hobbies try to claim deductions bc the intend to make a profit. Isnt this a hobby of Nickerson? Income from a hobby is reportable and not deductible.

183 v. 162: Hobby v. Trade or business

Whether a home is a business or a personal expenditure.

HOME OFFICE - BUSINESS OR PERSONAL EXPENSE?


280A DISALLOWANCE OF CERTAIN EXPENSES IN CONNECTION W/ BUSINESS USE OF HOME, RENTAL OR VACATION HOMES, ETC. Basic Rule: No deduction for second homes or any dwelling unit used by the taxpayer as a residence Exceptions Can deduct for certain business or rental use 280A(b) o When homes are used EXCLUSIVELY and on a REGULAR BASIS for PPB for any trade or business For use by patients or clients in the normal course of business In the case of a separate structure in connection with trade or bus. The statute does not say it has to be a primary business

What is a trade or business? How does it differ from a profit making venture that is not a trade or business.

18

Moller v. US (pg 473) TP is retired and spend days managing their money trading stocks. They want to deduct the cost of their home office under 280A. Held: No deduction bc they were not engaged in trade or business under 280A(c) o Is the room exclusively used? Yes o Regular basis? Yes o The issue is whether being an investor is a trade or business under 280A(c)(1)(A) Court draws a distinction btw active and passive investors o Although these people are investors, they are not an active business no active day trading Seems they are passive

IRS v. Soliman MD works in many hospitals so he doesnt have a proper office. He does admin. work in a home office and wants to deduct the expenses under 280A Held: No deduction bc not his principle place of business o 280A(c) Exclusively Yes Regular Yes Is it the principal place of business? This is the issue. If he rented a space in an office building, there would be no question that he would get a deduction

Soliman Overturned by Congress A paragraph to 280A(c) in December 1998 in response to Soliman that overturned the courts decision. This addition could also have interpretive issue PPB includes a place of bus. which is used by the TP for the administrative or management activities of any trade or business of the tp if there is no other fixed location of such trade or business where the tp conducts substantial administrative or management activities for that bus. Today, Soliman would get the deduction.

212 EXPENSES FOR PRODUCTION OF INCOME Expenses for passive collection of income (e.g. newspapers) are DEDUCTIBLE Deduction is allowed for the orginary and necessary expenses for o Production or collection of income o Management, conservation or maintenance of property held for the production of income o Expenses in connection with determination, collection or refund on tax

LEGAL EXPENSES FOR DIVORCE


US v. Gilmore (pg 538) W and H are getting a divorce. W wants Hs stock. He wants to deduct his legal expenses for fighting to keep his stock. H claims a deduction under 212. Held: NO DEDUCTION bc the origin of the dispute was personal Does he qualify under 162? What about 212(2)? TP claims he was conserving his property. Legal fees to hold onto your business are different than legal fees for the divorce.

Henderson v. IRS (pg 485) Attorney General bough paintings to decorate her office and deducts expense under 162 ordinary & necessary business expenses Held: No deduction o These were non-deductible personal expenses under 262

19

TRAVEL, ENTERTAINMENT, ETC. EXPENSES & DEDUCTIONS


274 DISALLOWANCE OF CERTAIN ENTERTAINMENT EXPENSES General Rule: NO DEDUCTION for entertainment, amusement or recreation activities Unless: Entertainment related activities were directly related to business Expense has to still meet 162 test of ordinary and necessary 273(a)(3) DENIAL OF DEDUCTION FOR CLUB DUES o No deduction for clubs o Maybe it became to difficult to determine administratively what was a legitimate business expense 274(d) SUBSTANTIATION REQUIRED o Must be substantiation of expenses to get deductions o Differs from 162, which does not require substantiation of ordinary or necessary business expenses o This was Congress attempt to address the problem 274(n) ONLY 50% OF MEAL & ENTERTAINMENT EXPENSES ALLOWED AS DEDUCTION o Can only deduct 50% of meals and entertainment o Tension btw policies of difficulty to administer on a case-by-case basis o If we have 274(n) do we need the earlier provisions? Zelinsky says good argument that there is no need for the earlier provisions like 274(a)

Boyd Gaming v. IRS Casino Ees cant leave premises during their shifts so they get free meals in the cafeteria. Casino wants to deduct cafeteria expenses. Casino says they are exempt from 50% limit bc the meals are a de minimus fringe (274(n)(2)(b). Held: 50% limits does not apply o Under 194(b)(4) Since more than half of the Ees were provided meals for the convenience of the ER, all the meals are considered to be furnished for the ERs convenience so its presumed that revenue derived equals or exceeds operating costs so the cafeteria is a de minimus fringe and 50% limit doesnt apply Interplay btw 119 (meals & lodging as income), 274(n) & 132 Map of analysis o 162(a) ordinary & necessary for trade or business o 274(n)(1) Boyd argues they are exempt from the limit under 274(n)(2) bc they get a full deduction if it is a 132(e) fringe Boyd argues it does fall under 132(e)(2) o 119(b)(4) If it qualifies under 119, then the ER is deemed to have qualified under 119 bc they have met his direct operating costs under 132(e) If it qualifies under 132, they get a full deduction under 274(n) Zelinsky says this turns out right

Moss v. Commissioner (pg 497) Lawyers in firm meet every day for lunch in a restaurant and want to deduct expenses under 162. Held: NO DEDUCTION bc not necessary bus. expense Today, under 274(n)(1), the deduction would be 50% Maybe if there were guests, deduction would have been allowed.

Danville v. US (pg 501) Company sent customers and their spouses to the Super Bowl and wanted to deduct under 162 as trade/business expenses. Held: Not deductible bc not ordinary/necessary expense (162) and not directly related to tps active business (274(a)) Where is the line btw what entertainment expenses are deductible or not?

274(m)(3) TRAVEL EXPENSES OF SPOUSE, DEPENDENT, OR OTHERS No deduction for spouse or dependent accompanying a taxpayer on business travel unless o Individual is an EE of the tp o The travel of the indiv. is for a bona fide business purpose, and o Such expenses would otherwise be deductible by this person Looks like codifications of Danville

20

Problems Pg. 501

162(a)(2) TRAVELING EXPENSES: ORDINARY & NECESSARY V. PERSONAL EXPENSES

Flowers (pg 513) TP lived in Jackson bud had main office in Mobile, so he traveled back and forth a lot. He wants to deduct his travel expenses under 162(a)(2). Held: Traveling expenses were not deductible bc it was the tps choice to live so far away from work Could argue that the statute says necessary and here it is necessary id doesnt say so necessary -- What degree of necessity should we take?

Hantzis (pg 518) Married women went to school and lived in Boston. She worked in NY for the summer and deducted her living expenses for the summer under 162(a)(2). Held: Not deductible bc not incurred while away from home o Court considered her home to be NY during the summer the place of her employment o Court uses 3 part test under 162(a)(2): Is it ordinary & necessary? Was it while she was away from home? Was it in the pursuit of trade or business? Definition of while away from home o Is Boston her home? o Is her home for the summer NY? (Maybe Boston is her domicile) o Can have more than one home. Argue she had 2 homes NY & Boston

Congress Response to Hantis Congress changed 162 they added that the TP is not considered to be temporally away from home if she is away for longer then 1 year.- BUT THIS DOES NOT NECESSARILY MEAN THAT IF PERSON IS AWAY FROM HOME FOR LESS THEN 1 YEAR, THEN HELL BE CONSIDERED TEMPORARILY AWAY FROM HOME AND CAN DEDUCT EXPENSE- ALL ITS SAYING IS IF MORE THEN A YEAR THEN DEFINATLY CANT DEDUCT EXPENSE.

Revenue Ruling 94-97 (pg 529)

CLOTHING EXPENSES
Pevsner (pg 535) Woman works in YSL and has to buy YSL clothes to wear at work. She wants to deduct cost under 162 as a business expense. Held: No deduction (But, tax court came out the other way) Court said she didnt meet the 3 part test: o Condition of employment (ER makes you wear it) o The clothing is not adaptable to general usage (this is so objective) o Clothing is not worn for general usage (army uniform) Is the test whether the type of clothes is for the benefit of the ER If ER requires clothes from a certain store? Is it a test of whether the EE has choice?

EXPENSES OF EDUCATION
Carroll v. Comm Cop decides to go to undergrad in hopes of being a lawyer. Wants to deduct his tuition. Held: NO DEDUCTION bc it was not a trade or business bc it was not directly related to improving his job skills Deductions must somehow improve his current job to be deductible-no direct relationship btw his coursework and being a cop -it was a personal expense- Perhaps if taking criminal law, etc. might get deduction otherwise nothing under 162 There is also an argument that education is a capital expense producing income over many years but that is speculative and difficult to administer

21

Krist v. Comm Educational trip of six months for 1st grade teacher- She says hardship b/c did not stay in best hotels and visited 5 schools plus had to give presentation to other teachers Held: Court dismisses and Congress ensured suits not repeat by adopting 274m2 disallowing deductions for educational trips

CHILD CARE EXPENSES


Smith v. IRS (pg 509) Child care as a deductible expense bc wife works. Wife wants a deduction under 162 for child care expenses bc its ordinary & necessary for her business. Held: Expenses for child care are not deductible is a personal expense under 262 This case was prior to 21

21 EXPENSES FOR HOUSEHOLD &DEPENDENT CARE SERVICES NECESSARY FOR EMPLOYMENT 21(a): o (1) General Rule: If a tp maintains a household w/ qualified dependents, a percentage of the amount spent on services to care for that dependent is used as a credit against taxes owed. o (2) Applicable Percentage Defined 30% reduced (but not below 20%) by 1 percentage point for each $2,000 by which the tps AGI for the year exceeds $10,000 Percentage goes down as income until it reaches 20% After 12/31/2002, changes to 35% 21(b) Qualifying individual o Broader definition than in Smith case Includes dependent or spouse if incapable of taking care for themselves Under the age of 13, or Physically or mentally handicapped and incapable of caring for himself (spouse or child) Smith would get a tax credit under this new statute 21(c) Amount of Credit (Amount on which credit is calculated) o Up to $2,400 for 1 qualifying individual o Up to $4,800 for 2 or more qualifying individuals Same cap no matter how many kids after 2 o These amounts are change after 12/31/2002

EXAMPLE: I have 2 kids under age 13. Spend $5,000 in daycare expenses. Salary is $10,000/year. o Under 21(c), the max amount of daycare expense that can be considered is $4,800. o Credit under 21(a)(2): Get full 30% (Take 30% of $4,800 = $1,440, this is the credit applied to taxes) o If my income was over $10,000, then the phase out applies: If I make $20,000, then the percentage goes down to 25% If I make $30,000 and over, the percentage falls to 20% CREDIT V. DEDUCTION 100,000 gross income -20,000 minus deductions 80000 taxable income tax rate x .4 32,000 tax credit -1,000 31,000 amount owed to the government

$1 credit is more valuable than $1 of deduction - $1 of deduction just reduces taxable income by $1, while $1 credit takes it off the amount paid to the govnt.

22

INTEREST
163 INTEREST 163(a) General Rule: Interest is deductible if paid or accrued within the year on indebtedness 163(d): LIMITATION ON INVESTMENT INTEREST (investment interest is like securities) RESPONSE TO TAX SHELTER o (1) Amount allowed as a deduction for the year cant exceed the net investment income of the tp for the year o (2) Carryforward: Amount not allowed as a deduction can be carryforward to the next year 163(h): DISALLOWANCE OF DEDUCTION FOR PERSONAL INTEREST (like credit card interest) o NO DEDUCTION o 163(h)(2): Everything is personal interest, except: (These things are deductible) Trade or business interest (Deductible w/o a limit) Investment interest (e.g. securities) (Deductible w/ a limit) Qualified resident interest (e.g. interest paid on mortgage is deductible w/ a limit) You can deduct this interest only on 2 homes (163(h)(4)(A)) 2 types of qualified residence interests: o Acquisition of Indebtedness 163(h)(3)(B) - (only up to $1M) This interest is from building, buying, improving, or refinancing a qualified residence. o Home Equity Indebtedness 163(h)(3)(C) Interest on any debt secured by the qualified residence is deductible. The amount of interest treated as home equity indebtedness cannot exceed $100,000.

Loan to repair house is good b/c improving and will raise basis cost of house However, w/ refinancing new basis is old mortgage principal Questions pg 449-450 1. To tax consequence for taking money out of the bank and buying a car; Interest on $50K loan is deductible bc under 163(h)(2) bc the proceeds go to a trade or business Suppose she had taken the $50K from the bank to buy the restaurant. In that case there would be no deduction under 163(h)(1). How she manipulates her funds determines the tax consequence. So, using loan to buy a business is a good tax consequence, while using her own money is not. 2. TP borrows against her home and uses the $50K loan to buy a car Under 163(h)(3)(c): She owns the residence, amt being borrowed does not exceed fair market value, amt she is borrowing is under $100K limit

So, she can deduct the interest from the loan. Why do we want to give a break to home owners rather than renters? Why are we rewarding home owners for borrowing money, but renters dont get a break for borrowing money? 221 INTEREST ON EDUCATION LOANS Rule: Interest paid on qualified education loans is DEDUCTIBLE but there is limit Limit: Maximum deduction for this interest is $2,500. o There is a phase out when tps income hits $50,000 Phase Out Formula: Excess of income over $50,000 (if income is $60,000, then excess is $10,000). Then take ratio of excess to $15,000 (10:15). Then apply that ratio to the deduction and you lose that much of the deduction.

Ex. Interest expense $2,000, AGI $65,000 Reduce amount over 60,000 resulting in 5,000 ratio to 15,000 is 1/3 -so lose a third of deduction

If their income had been 75,000 would be no deduction. 0 to 15,000 is zero. Liberalized starting this year to 50,000/100,000 hurts areas of higher income areas

23

Ex. Using OLD numbers: Salary is $50,000/year. The excess of $50,000 over $40,000 is $10,000. The ratio of $10,000 to $15,000 is 2/3. So, I lose 2/3rds of the deduction. 2/3rds of $2,000 (amt allowed for the deduction) is $667. So deduction would be $2,000 - $667 = $1333. DEFINITION OF QUALIFIED EDUCATION LOAN o Indebtedness incurred by the tp to pay higher education expenses for the tp, their spouse or dependent o Education loan can only last for 5 year

TAXES
164 -- TAXES The following taxes are deductible: o State, local, & foreign real property taxes o State & local personal property taxes (e.g. car) o State, local & foreign income, war profits, and excess profit taxes These things are NOT DEDUCTIBLE: o Sales tax o Federal income tax o Social security tax o Federal estate and gift tax o State inheritance tax NOTES: o Any state, local or foreign taxes paid or accrued in carry out a trade or business or a 212 activity (production of income) is DEDUCTIBLE (e.g. excise, import, social security on EES, gas, etc.) o Any tax paid or accrued on buying or selling a property is treated as part of the cost (if you are buying) or reduction of the amount realized from a sale

24

DEPRECIATION DEDUCTIONS
Certain assets are depreciated you get deductions over time each year you deduct the amount of the asset thats been used up that year. ADJUSTED BASIS: Each year, when you take a deduction, the basis (cost) of the asset is decreased by the amount of the deduction. 1016 Accounting Depreciation: Its better to do rough and in approximate estimates o Estimate Useful Life o Estimate the Salvage Value (value at the end) o Allocate the decrease used over the life at an equal basis (Straight-Line)

Example: Buy equipment for $1,000 Useful life = 9 years Salvage value=$100 So, you are using up $100 per year (1,000-100=$900/9=$100) 167 - DEPRECIATION General Rule: Depreciation Deduction for wear & tear of property used in business or held for the production of income (not personal expenses) Things NOT DEPRECIABLE o Land, securities, corporate stock, goodwill, property not used for trade or business For land, no useful life (potential dispute btw tp and govnt is a building sold w/ land govnt wants to allocate to the land, while tp wants to allocate to the building)

168 ACCELERATED COST RECOVERY SYSTEM (ACRS) 1981: Congress switched to the ACRS (Pres. Reagan changed things) An accelerated system designed to reward business by an early depreciation deduction Key Features of ACRS o No longer a system of salvage value 168(b)(4): Allows depreciation of the entire amount Gives a greater deduction; eliminates a category of disputes btw tp and IRS o No longer depreciate based on projected useful life, rather we group things together by useful lives and let them be depreciated over a set time by 10 categories 168(c) Allows greater and earlier deductions Gives businesses incentive to make deductions o Depreciate more rapidly over a shorter period of time (so a property w/ 23 year useful life can be depreciated over 15 years) o Allows many forms of property to be depreciated on a declining balance method rather than a straight-line method 163(b) Some things are still depreciated by straight-line (like real estate) 168(a): Tangible property is depreciated 168(b): Depreciation Methods (1) 200% Declining Balance (Double Declining): i. For equipment. ii. Each year, deduct double what you would under straight-line. iii. Then switch to straight-line when it yields a larger deduction. (2) 150% Declining Balance i. Used when: 1. Property is a 15 or 20 year property 2. Property is used for farming bus. 3. If tp elects to do so (3) Straight-Line: i. Nonresidential real property ii. Residential rental property iii. Railroad, water, utility iv. 168(c): RECOVERY PERIOD o The applicable recovery period (# of years over which the asset is depreciated) is determined by this chart 168(d): APPLICABLE CONVENTION o applies when equipment is purchased not on the 1st of a period 168(e): CLASSIFICATION OF PROPERTY o This chart assign useful lives to property

25

www.swapnotes.com E.g. property shall be treated as a 3-year property if it has a life of 4 or less years.

Example 1: Have equipment that will last for 19 years. Purchased for $2,000 and have a salvage value of $100. 167 Accounting $1,900 (salvage value) 19 lives Straight-line basis 168 ACRS $2,000 (no salvage) 10 years (class life) 200% declining balance

Depreciate Useful Life Method Result

$100 each year for 19 years

$400 (20% of $2,000) for 1st Year

Over time, the deductions will be the same, but the timing is radically different bc early on the ACRS method allows bigger deduction. 2nd year

$100

Double Declining Balance Balance= $1600 Double 10 years: X 20% $320

Example 2: Buy piece of equipment for $1,000. 5 year class life. 200% declining balance rule 40% depreciation in year 1 Year 1 Year 2 Year 3 Deduction of $400 ($1,000 * 40%) New basis becomes $600 Deduction is $240 ($600 * 40%) New basis becomes $360 168(b)(1)(B) Switching to the Straight-Line Accelerated Deduction = $144 New Basis is $216 Straight-line Deduction=$120 So, this isnt the year to switch to straight-line bc accelerated is still more

Year 4

Accelerated Deduction= $67 (40% * 216) Straight-Line = $108 This is the year to switch to the straight-line

Year 5 Straight-Line=$108 Sum of all deductions equals $1,000 Basis after 5 years is $0. But, this is a fiction bc obviously property lasts and doesnt disappear

So, if someone offers you $150 for the property. 1001: Gain=Amount realized - adjusted basis So, there would be a gain of $150 Artificial ???? (what is this called?) If it was sold at the end of year 3 for $300, the gain would be $300 - $216=$84 (1001)

1016(a)(2) ADJUSTMENT TO BASIS

Simon v. Commissioner (violin bows) TP wants to depreciate antique violin bows. Says they use they bows for their trade or business (167(a)) Held: DEPRECIABLE Is this right? o The bows last forever (like land) it will always be an antique even if it cant be used to pay o As dissent says, Congress did not intend to get rid of the useful life requirement (Zelinsky find this persuasive) o The bow is actually increasing in value as time goes on o 168(e) limits deduction to things that have a useful life

26

Everson v. US (bushes as windbreaks) TP bought farm and wanted to depreciate trees that were windbreakers. Held: NO DEPRECIATION DEDUCTION for bushes and trees Step 1: Land is not depreciable (no useful life) Step 2: This improvement to the land does not have a useful life

DEPLETION OF NATURAL RESOURCES (we wont focus on thisjust know there is a form of this) Minerals, oil & gas can be used up Same idea as depreciation deduction 2 different historical concepts o Recoverable Units 611 Estimate on basis on how much is there. If there are 1,000 gallons of gas and you pay $1,000 for it, each gallon that comes out is worth $1 and you get a deduction for each gallon that comes out o Percentage Depletion 613 Get a deduction of percentage of amount sold There is a big debate on which method to use

DEPRECIATION DEDUCTIONS FOR INTANGIBLE ASSETS Goodwill: name of business, customers, location something besides tangible assets that adds value to business

Newark Morning Ledger Tp bought newspaper company and wants to depreciate (deduct) for the paid subscribers list under 167. Expert testifies that the customer list has a useful life of 23.4 years Held: It is depreciable o If you look at each individual subscriber (a specific set of names) it is true that they will expire so it has a useful life Names become obsolete so depreciable under 167 o But, if we look at the list as an entirety, it is regenerated and can last forever so no useful life Then it couldnt be depreciated o Maybe consider the list as it is now over time they do get new people, but the original people have a useful life Is this list like land or like a building? Minority says this is just goodwill Majoritys test for determining whether the intangible is depreciable: is fact-based taxpayer must prove it

Congress responded w/ 197.

197 AMORTIZATION OF GOODWILL & CERTAIN OTHER INTANGIBLES Amortization means depreciation for intangibles, Added goodwill to depreciable intangibles 197 is the analog for intangible property to 168 for tangibles 197(a) General Rule: All intangibles are depreciated by straight-line method over 15-years o So, a patent w/ 3 years left gets depreciated over 15 years o If useful life is less than 15 years, you get 15 years so it is worse for the company o If the intangible has a longer useful life than 15 years, it is better for the taxpayer o Could argue this eliminates any factual disputes 197(d): Intangibles include: o Goodwill o Going concern value o See Code for list of others 197(e) : EXCEPTIONS Items not depreciable: o Land, corporate stock, etc.

27

Zelinskys Method Decision Tree

Personal Expenditures: Non-deductible under 262 except for things like mortgage interest Trade & Business: o 162 expenses are deductible o Capital Expenditures 263: Not deductible, but may be depreciable; If something is an expense, it is not deductible, but might be depreciable Tangibles Depreciable: Machinery, equipment, etc. Non-depreciables: land Intangibles Some are not depreciable: corporate stock, etc. 197 depreciable intangibles Goodwill is now in this category 167 intangibles covered by useful life rule like computer software (167(a)(3)

Self-created intangibles (patents, franchises, etc.) can be depreciated. 197(c)(2) 197(d)(c) self-created patents. Hypo: Pfizer makes a patent. Can salaries of scientists be deducted? -- 263A Deduct these expense Can salaries of people who make a film for Pfizer be deducted? -- 263A(b) not deductible, but can be capitalized bc its tangible If Pfizer buys a patent? -- 197 amortizable over 15 years Frontier Chevrolet Co. Frontier is owned by 2 people: Roundtree owns 75% and Mr. Mendhold owns 25%. Company buys its own stock back from Roundtree, so Mr. Mendhold now owns 100% bc he is the only shareholder. They sign a noncompete - Frontier wants this agreement to be a 167 depreciable. Issue: Was the covenant not to compete entered into in connection w/ acquisition of the business? o If yes, it has to be amortized over 15 years under 197 This is bad for him bc it only lasts 5 years o If no, he can depreciate it over the useful life Holding: TP loses Must amortize over 15 years

CHARITABLE DEDUCTIONS
170 CHARITIABLE CONTRIBUTIONS & GIFTS 170(a): General Rule: Charitable contributions are deductible when a payment is made, not when you make a pledge. o Also, remember you must give w/ a detached disinterested generosity 170(b): LIMITS ON THE PERCENTAGE o Contributions cant exceed 50% of contribution base (adjusted gross income) means cant deduct more than 50% of adjusted gross income (170(b)(1)(F)) - for contributions to churches, educational organizations, hospitals, public organizations, etc. o Can deduct up to 30% for contributions to other charities 170(c): Charitable contribution means contribution or gift to or for: o A State, possession of the US, etc. o Corporation, trust, etc. o War veterans o Fraternity (lodge, Bnai Brith) o Cemetery company 170(d): Any part of the contribution that is above the 50% limit is carried over to the succeeding 5 years 170(f)(8): Requirement that contributions be acknowledged by the charity (substantiation) (Similar to the anti-fraud provision of meals & lodges)

MAIN POINT: Deduction allowed for charitable contributions when made w/ a detached and disinterested generosity. The contribution must be to a non-profit and non-political org. The deduction is only good if you take itemized deductions.

28

Ottawa Silica v. US (pg 426) TP is a silica mining company in CA. He donated the land to the community to be used as a school. His property would increase in value and access to his land would increased. Holding: No deduction for the charitable contribution bc donor was getting a substantial benefit. Statutory basis: o 170 (c) contribution or gift this is not a gift [but is it a contribution?] o 170(c)(1) exclusive public purpose

Bob Jones University v. US (pg 436) Bob Jones is a Christian University that doesnt admit non-Christian students and doesnt allow interracial dating among its students Holding: No deduction for contribution to this racially discriminatory school bc its against public policy. o 170(c): Is this a charitable contribution? Can argue ct. was wrong bc the Code doesnt have a public policy provision and its a religious educational organization, so tp should get the deduction.

If we take Bob Jones seriously, the Catholic Church would not be a charitable organization.

DEDUCTIONS FOR MEDICAL EXPENSES


213 MEDICAL, DENTAL, ETC. EXPENSES 213(a): General Rule: deduction for expenses for medical care not compensated by insurance for the tp, spouse or dependents to the extent such expenses exceed 7.5% of AGI 213(b): LIMITATION WITH RESPECT TO MEDICINE & DRUGS o Can only deduct for prescribed drugs or insulin 213(d): DEFINITIONS o (1) Medical Care means amounts paid for the diagnosis, cure, mitigation, treatment, or prevention of disease TEST OF PURPOSE OF CARE o (2) PAYMENTS FOR LODGING AWAY FROM HOME FOR TREATMENT Deductible if the care is provided by a MD in a licensed hospital AND there is no significant element of personal pleasure, recreation or vacation in the travel o (9): medical care does not include cosmetic surgery unless the surgery is necessary to ameliorate a deformity arising from or related to a congenital abnormality, an injury resulting from an accident or a disease E.g. Cost for braces Not deductible bc cosmetic? Cosmetic surgery for a movie star? Is it capital under 162?

Taylor v. IRS (pg 416) TPs doctor told him not to mow the lawn so he wants to deduct the cost of hiring a gardener to low the lawn as a medical expense. Held: NOT DEDUCTIBLE bc its a personal expense under 262 Today, it is unlikely this would get a deduction bc of the current rule that expense must be over 7.5% of AGI Test under statute is purpose

Ochs v. IRS (pg 417) Sending kids to boarding school bc mother is sick. Holding: Not deductible bc its a personal expense. But, if wife went away, it would be deductible as a travel for medical care.

29

DETERMINING THE AMOUNT OF TAX OWED


Progressive Tax System 5 brackets dependent on how much income you have o More income, higher rates o Rates will go down according to the 2001 act by Congress over the next 9 years o Bushs tax cut will affect 1 Different filing brackets for status o Joint filing o Head of households o Unmarried Individual o Separate filing for married people o Trusts and Estates o Corporations (11)

Hypo: Tax payer w/ no dependents (no spouse or children) 3. Gross Income 90,000 Wages 10,000 Municipal Bond Interest 5,000 Bank Interest 5,000 On-Premise Housing Gross Income = $95,000

Income 61 Not Income 103 Income 61 Not Income 119

4. Deductions 20,000 Rent 10,000 Food 10,000 Charity 5,000 State Taxes Deductions = $15,000

No deduction 262 No deduction 262 Deductible 170 Deductible 164

5. Personal Deductions $2,000 151(d)(1) Total Deductions = $17,000

Taxable Income = $78,000 6. In the hypo, the TP is an unmarried individual in the third bracket Tax = $12,107 plus 31% of the excess over $22,100 63 TAXABLE INCOME DEFINED 63(a): Taxable Income is gross income minus deductions 63(b): INDIVIDUALS WHO DONT ITEMIZE THEIR DEDUCTIONS 63: STANDARD DEDUCTIONS o (2) BASIC STANDARD DEDUCTIONS (A) $5,000 joint returns (B) $4,400 for head of household $3,000 for individual (D) $2,500 for married indiv. filing separate return o (4) ADJUSTMENTS FOR INFLATION 63(d): ITEMIZED DEDUCTIONS o Deductions allowable other than (1) to arrive at AGI or (2) personal exemptions under 151 63(e): ELECTION TO ITEMIZE o Congress has given taxpayers the option to skip the deductions and electing the standard deduction. 70% of all taxpayers elect the standard deduction o These people elect it bc its more than their actual deduction o TPs then dont have to keep track of their outlays and the IRS doesnt have to audit you o Another way to look at it is to see the standard deduction as promoting the progressiveness of the tax system.

30

62 ADJUSTED GROSS INCOME DEFINED Adjusted gross income (AGI) = gross income minus certain 17 deductions o Trade or business deductions, Alimony, etc. This value is important bc many deductions are based on AGI o 170 charitable deductions Taxpayer can still take the standard deduction under 63 if they claim these special deductions under 62 (can take standard deduction and a deduction for alimony) (if take the standard deduction, cant take deduction for things like medical expenses) Charities are always fighting to be moved into 62 deductions so that they can tell donors that they will get a deduction and also be able to take the standard deduction.

[In our hypo, the AGI and taxable income are the same.]

Zelinskys Method Gross Income (61) 62 (above the line deductions)

AGI Itemized Deductions (Both ways get the personal exemption 151)

Stnd Deduction or (63) Taxable TAXES - Credits

Net result btw whether deductions are taken to get to AGI or Taxable, its just a timing issue. Decision Tree: - determine GI (61) - Then take 62 deductions to get to AGI (above line deductions) - Then, under 63, you have a choice of 2 ways to get to taxable income: Deduct 151 personal exemptions and standard deductions OR Deduct 151 personal exemptions and itemized deductions. - now AGI - itemized or standard deduction(whichever is larger) and now subtract that number with personal exemptions - now you have your taxable income - take that amount and look at 1 to apply rates - now u have the amount u owe in fed taxes - but now u apply the credits u have (21) to reduce the amount of taxes u owe.

31

CREDITS
24: CHILD TAX CREDIT Per child, you get this credit plus the exemption This credit only applies to a qualifying child dependent child under age 17 24(a): Credit against tax for each qualifying child 24(b): LIMITATION BASED ON ADJUSTED GROSS INCOME o Phases out as income grows o Reduced by $50 for each $1,000 by which the tps AGI exceeds the threshold amount o 24(b)(2): THRESHOLD AMOUNT $110,000 for a joint return $75,000 for indiv. not married $55,000 for married indiv. filing separately 24(c): QUALIFYING CHILD

25A: HOPE AND LIFETIME LEARNING CREDITS 25A(a): ALLOWANCE OF CREDIT: TP can get the Hope Scholarship credit and the Lifetime learning credit (1 student cant get both, but a parent with 2 kids can get both 1 for each kid) 25A(b): 25A(c) 25A(d) 25A(f)(1)(A) 25A(h)

32: EARNED INCOME Usually for low-income w/ children Based on social policy gives a targeted relief to a specific group Phases out as income grows 32(a): ALLOWANCE OF CREDIT 32(b): PERCENTAGES AND AMOUNTS

FEDARAL INSURANCE CONTRIBUTION ACT (FICA) 3111 & 3121 Tax solely on wages Poor people get hit hard bc they dont get exemptions and standard deductions that they get on income tax SS tax is a flat tax So, poor people wont pay income tax, but they will pay FICA on their SS

32

PERSONAL EXEMPTIONS
Way of guaranteeing people that their basic cost of living money wont be taxed Keeps poor people from paying tax

151 ALLOWANCE OF DEDUCTIONS FOR PERSONAL EXEMPTIONS 151(a): ALLOWANCE OF DEDUCTIONS FOR EXEMPTIONS PROVIDED 151(b): TAXPAYER AND SPOUSE o Get a deduction for both 151(c): ADDITONAL EXEMPTION FOR DEPENDENTS o Get a deduction for each dependent if: Dependents GI is less than the exemption amount, OR Dependent is a child of the TP who Is under 19, or Is a student under 24 151(d): EXEMPTION AMOUNT o $2,000 o Phaseout: Phased out at the rate of 2% for every $2,500 that the TPs AGI exceeds the threshold amount o Threshold Amount: $150,000 for joint return $125,000 for head of household $100,000 for indiv. $75,000 for married filing separately 153(e): REDUCTION OF PHASEOUT o 2006 and 2007, have only 2/3 of the phase out o 2008 and 2009, have only 1/3 of the phase out

Ex: AGI=$152,500 Threshold amount -$150,000 $2,500 can only take 98% of exemptions

AGI=155,000 150,000 Threshold $5,000 Can only take 96% of exemptions (lose 4%)

4% of $2,000 = $400, so the family looses $400 from its deductions 152 DEPENDENT DEFINED Dependent Status 152(a) o See Code for list of 9 types of dependents o Tp must provide more than of the support o No requirement that dependents live w/ tp for 1-8 o Must live w/ tp under number 9 152(d) o Scholarships dont count toward the of support test 152(e)

Miller v. IRS TP claimed deduction for their kids but they dont want to put SSN bc of religious beliefs Held: Must have ids to get a deduction for dependents 151(e): Each dependent must have a TIN This requirement was to prevent fraud

67 2% FLOOR ON MISCELLANEOUS ITEMIZED DEDUCTIONS (see other outline pg 38) Creates a category of miscellaneous itemized deductions These deductions allowed only if the aggregate of these expenses exceed 2% of AGI Classic case is an employees deduction under 162 Can no longer deduct the Wall Street Journal
s.com

33

68 OVERALL LIMIATION ON ITEMIZED DEDUCTIONS (other outline pg 38) Phase out of all of deductions based on the gross income The higher your AGI, the less itemized deduction you get If a persons AGI exceeds $100,000, his itemized deductions will be reduced by the lesser of: o 3% of excess of AGI over $100,000, or o 80% of itemized deductions otherwise allowable.

Ex: Married couple has AGI of $110,000, which is $10,000 over the applicable amount. So, the itemized deduction amount is reduced by: 3% of $10,000 = $300

1 TAX IMPOSED Rates for: (a) Married filing jointly (b) Head of household (c) Umarried indiv. (d) Married indiv. filing separately (e) Estates & trusts (g) Kiddie Tax Child is taxed at the parents rate if this section applies to them for unearned income Ex: Child has a large trust fund from grandparents, but parents have no income Child gets taxed at the high tax bracket

34

EXECEPTIONS TO THE RULES: NONRECOGNITION OF GAIN/LOSS


BASIC RULE: 1001 gain/loss is recognized upon the sale or disposition of property There are 3 major exceptions: o Sale of principle residence - 121 o Like-Kind Exchanges - 1031 o Involuntary Conversions - 1033

1. SALE OF PRINCIPAL RESIDENCE


121: EXCLUSION OF GAIN FROM SALE OF PRINCIPAL RESIDENCE Selling your home is tax free; exclude all of the income from taxation 121(a) GI does not include gain from sale or exchange of property if, during the 5-year period prior, such property has been owned and used by the tp as the tps principal residence or periods aggregating 2 years or more (This means that in the last 5 years tp had to live there for at least 730 days) 121(b): LIMITATIONS o (1) Amount excluded cant exceed $250,000 (must report amounts over $250K) o (2) The limit is $500,000 for joint returns o (3) This non-recognition gain applies to only 1 sale/exchange every 2 years.

2. LIKE KIND EXCHANGES


1031 EXCHANGE OF LIKE-KIND PROPERTY 1031 is called a rollover provision transferring from one investment to another General Rule: No gain/loss shall be recognized on the exchange of property held for productive use in trade or business or for investment if such property is exchanged solely for property of like kind. o Mandatory provision (shall) o Personal property does not qualify it must be for trade, business or investment o Most like kind exchanges involve real estate This provision DOES NOT apply to CASH -- Differs from 121 in which cash is not taxable 1031(a)(2); EXCEPTIONS: These things are NOT like-kind o Stock in trade or other property held for sale o Stocks, bonds, or notes o Other securities or evidences of indebtedness or interest o Interests in partnerships o Certificates of trust or beneficial interests o Choses in action (right to sue someone)

Revenue Ruling 82-166 Exchange of gold for silver Holding: Not like kind Does it matter how the property is used? What if they are interchangeable for some uses and not interchangeable for others does it matter? Are we looking at the investment qualities, jewelry qualities, etc.?

1031(a)(3); REQUIRMENT THAT PROPETY BE IDENTIFIED AND THAT EXCHANGE BE COMPLETED NOT MORE THAN 180 DAYS AFTER TRANSFER OF EXCHANGED PROPERTY Congress overturns Starker. o If you want to do a like-kind exchange but havent found the property you want yet, in order for the exchange to be considered like-kind, the property u want must be identified w/in 45 days after the day you transfer your property and the exchange must be completed after the earlier of 180 days after tp transfers his property, or The due date for the tps tax return (if that is earlier)

35

Starker v. US (prior to enactment of 1031(a)(3) TJ has timberland. Crown is a paper company and they want Starkers trees. Crown doesnt want to wait until TJ can find another parcel to exchange bc they want the land now. TP swaps land for a credit and they have 5 years to decide what they want to buy. TP says this is a like-kind exchange. IRS says like-kind exchanges implicitly means simultaneous exchange. Held: LIKE KIND EXCHANGE

HYPO: Jan 1: Starker give property to Crown. To avoid taxation, must meet these 2 provision: 1031(a)(3)(A): Crown must identify land to exchange by Feb. 14th 1031(a)(3)(B): By July 1st, must close on the property

This doesnt really fix policy concerns of the escrow in Starker.

Jordan Marsh (pg 276) TP is a property owner who sold 2 parcels of property for cash he had a loss on the sale. He then leased the property on a 30-year lease. Sale-lease-back: JM sold the property and got $ and a lease (probably sold to a financial institution who had the cash and wanted to get a stream of income). JMs position: This was a sale of property for cash (lease is separate) so not like-kind and he suffered a loss so its deductible; IRSs position: Exchange of building for a building and a lease like-kind exchange so not deductible. Held: Transaction was a sale, not an exchange. NOT LIKE-KIND. Loss is recognized now and deductible.

1031(b): GAIN FROM ECHANGES NOT SOLEY IN KIND - BOOT BOOT: property not of like kind exchanged in conjunction with like kind property. You are taxed on the gain you receive from boot. Ex: $1 basis in an apartment building; FMV=$10 o If sold for cash: $9 gain o Swap for a $7 apartment building? Wouldnt do this unless the person gives you $3 cash in addition to the building $3 to boot o In the example above, gain recognized is $3 This makes sense bc the $3 is liquid so taxes can be paid 1031(C) LOSS FROM EXCHANGES NOT SOLEY IN KIND BOOT o Not recognized

36

3. INVOLUNTARY CONVERSIONS
1033 INVOLUNTARY CONVERSIONS Involuntary conversions occur when tps property is condemned and she received an eminent domain award or if it is destroyed or stolen and she receives an insurance payment. The Code allows tp not to recognize such gain since it is unwanted. 1033(a): General Rule: If property (as a result of its destruction in whole or in part, theft, seizure, requisition, or condemnation) is compulsorily or involuntarily converted: then o 1033(a)(1): No gain will be recognized for conversion into similar property (similar or related in use or service) BUT, LOSS CAN BE RECOGNIZED o 1033(a)(2): Gain will be recognized for conversion into money or other non-similar property UNLESS: The Tp during the period specified in B, buys similar property if he does this, then gain shall be recognized only to the extent of boot (difference btw similar and non-similar properties). So, if tp puts cash back into similar property, the gain wont be recognized, except to the extent of boot The similar property must be acquired after the involuntary conversion TP is deemed to have purchased similar property if unadjusted basis = cost Period w/in which the property must be replaced till the end of 2 years. 1033(g)(1): CONDEMNATION OF REAL PROPERTY HELD FOR PRODUCTIVE USE IN TRADE OR BUSINESS OR FOR INVESTMENT o SPECIAL RULE: If real property held for productive use in business compulsorily or involuntarily is converted, property of a like kind to be hled either for productive use in business or investment shall be treated as property similar or related in service or use o LIMITATION: Doesnt apply to stock

EXAMPLES: If basis is $1, FMV is $10 and it is stolen. o 1001: $9 gain (taxed) o 1033: Involuntary disposition triggers tax Tax on $9 When you cash out involuntarily and reinvest it, there is no gain o Ex: govnt takes your land and gives you land somewhere else 1033(a)(1) If the conversion is to money, there is tax unless o If you reinvest the money w/in 2 years, no tax 1033(a)(2) Insurance proceeds fall under this provision To qualify for tax-free treatment look to 1033(a)(2)(A) o Reinvestment must be in property that is similar or related in service or use 3 Rollover Provisions (tax free) 121: o Flat permanent forgiveness sell your principle residence, get cash dont reinvest, pay no tax o Can cash out 1031: Like-kind o Property for property of like-kind doesnt generate take o Its not a permanent forgiveness its a postponement of tax if there is eventually a sale, it is taxed o Carry-over basis regime o Cant touch cash 1033 o Can get cash, but must reinvest o Not forgiveness, just postponement until tp sells new property

37

BASIS
1031(d): BASIS for LIKE KIND EXCHANGES New property acquired in the like kind exchange shall have the same basis as the old property

1033(b): BASIS OF PROPERTY ACQUIRED THROUGH INVOLUNTARY CONVERSTIONS New property has the same basis as the old property

HYPO: Basis=$1; FMV=$10 Government gives you $10 for your house. Amount realized is $10. Gain is $9. If you spend entire $10 on a new property similar, there is no tax. Basis of the new property is $1 (See 1033(b)) o Same basis idea for like-kind ideas 1031(d) Swapping 2 properties w/ value of $10, basis of your first property was $1 New basis in property is $1 If you only use $8 to buy a new house, you have $2 boot and are taxed on that amount - $2 is recognized (1033(a)(2)(A))

With either transaction, you have gain to extent of the boot Boot can also be property, rather than money

BOOT & BASIS


Boot (keep cash) No questions of boot under 121 Boot is an issue under 1033 bc you can keep some of the cash Keeping cash gives you income

1031(b): If an exchange would be like kind if not for the fact that boot is also being given, the recipients gain is equal to the boot. 1031(d): The basis for the new property, decreased by money received, and increased by the amount of gain (boot) tp has recognized

FORMULA FOR BASIS OF LIKE-KIND PROPERTY: 1031(d) Basis of original property FMV of boot + Gain recognized HYPO: Suppose you give your $10 building and receive $7 apartment building (like-kind), a $3 car (not like-kind). o $3 gain recognized 1031(b) o Need 2 basis (1031(d)): $1 + $3 (gain) = $4 must allocate this basis btw the 2 properties o Car = $3 (if you then sell the car for $3, there is no tax bc you already paid it, except on any appreciation) o Building =$1 (if you then sell the building, the tax you pay is on $6) HYPO: Basis=$1; FMV=$10; Get $3 cash and a $7 building o $3 of boot is $3 of income which is recognized o Basis in like-kind apartment is $1-$3+$3= $1 (see formula above) HYPO: o o o Basis=$1; FMV=$10; Get $1 cash, $2 car and a $7 building Actual gain (if this was a taxable transaction): $9 Gain recognized: $3 (1031(b)) Need 2 Basis: Allocate $3 o New Apartment: $1 o Car: $2 (could also say this is the FMV that was already taxed)

HYPO: Land Exchanged for Initial cost basis $2 Now worth (FMV) $12 $8 land, $3 plane, $1 cash o Gain recognized (1031(b)): $4 o Basis (1033(d)): $2-$1+$4=$5 in basis to allocate o Plane: $3; Land: $2

38

Always get the right result boot is taxable, boot has basis of its FMV bc it was taxed, and the like-kind property will always have the carryover basis bc the additional tax that we postponed will be recognized.

1259 CONSTRUCTIVE SALES TREATMENT FOR APPRECIATED FINANCIAL POSITIONS If someone owns an appreciated financial position and entered into a short sale of the property, this is considered a sale, and the gain is therefore recognized and taxed Constructive Sale o Short Sale: borrow something and sell it sell something that is not yours This works if the price goes down bc you replace and have profit left over. This is risky bc the price can go up and you sold it at a lower price and must buy back at a higher price Dont know if youve made money on a short sale until your buy the replacement IRS took the position that a short sale isnt realized until the transaction is over o Short Against the Box: Selling short when you already own the property borrow from someone else the same property you already own.

HYPO: Estee Lauder owns stock in her company w/ very low basis of $1. FMV now is $110. Lauder thinks this is a high value for the stock so might want to sell. If she sells, she pays tax on her $109 gain. If she holds on (shes 90), her kids will get her step-up basis when she dies. She borrows shares from Merrill Lynch and immediately sells them at the $110 value. Doesnt matter what happens to the price of the shares bc the shares she already owns she gives back to Merrill Lynch. She agrees to repay the shares to Merrill Lynch after her death. The $110 price is locked in for Lauder and her family. Short Against the Box. *There is no gain recognized so no tax paid bc she has not sold her shares like a wall btw her transaction. 1259 answers this. Put Option: Option to sell. Lauder says to Merrill that she will pay them for a put option, the right to sell at $110. If the market price goes down, she exercises her right to sell at the $110 price. If the market price goes up, then she doesnt exercise her option. 1259 answers this.

1259: Became effective for transactions made after 6/8/1997 (before this date, Lauders stock and the stock she borrows are different transactions). This provision changes the rules and doesnt allow Lauder to make such a transaction. This new section looks at the substance rather than the formality of the transactions so if it looks like you cashed out, you are taxed 1259(a): If there is a constructive sale of an appreciated financial position: o (1) the tp shall recognize gain as if such position were sold, assigned or otherwise terminated at its FMV o In other words, Gain is recognized anyway if you have a constructive sale you own it and either sell it short simultaneously or lock in the price w/ an option. 1259(b): Appreciate financial position owning something that there would be gain on if it was sold. o (1)Appreciated financial position = any interest position with respect to any stock, debt instrument or partnership if there would be gain if such position were sold, assigned or otherwise terminated at its FMV o (3) POSITION: Interest, including futures or forward contract, short sale, or option 1259(c)(1): CONSTRUCTIVE SALE o TP is treated as having made a constructive sale of an appreciated financial position if she: Enters into a short sale of the same property, etc.

39

WHO IS THE TAXPAYER?


Person who receives the income Person who generates the income Person who has dominion and control over the income?

1. EARNED INCOME
INCOME FROM SERVICES DIVERSION BY PRIVATE AGREEMENT Lucas v. Earl (pg 660) Husband and wife had a contract to be joint tenants on all income that husband produced. Earl (H) only thought he should be liable for tax on his salary and his wife should be taxed for the other . Held: H is the taxpayer and is responsible to pay tax on entire salary. o H is the service performer and therefore he is the taxpayer o Holmes says earned income should be taxed to those who earned it. o Fruit=income, Tree = Source Income should not be attributed to a different tree than that which it grew

INCOME FROM SERVICES DIVERSION BY OPERATION OF LAW Poe v. Seaborn (pg 663) TP and W lived in a community property jurisdiction, so of what he earns is hers by law. Held: They are taxed 50% each bc it was the law that split the income, not a private agreement o Husband and wife can file separate returns each w/ of the community income o TP= owner of property rights Opposite to Lucas v. Earl o This case is based on state law (spouses are co-owners), while Earl was based on a contract.

Congress eventually changed the decision. Today, husband and wife file joint returns so all income is pooled and it is close to each earning . Armantrout v. IRS (pg 671) ER implements a plan giving education benefits to certain Ees children. TP argues that their children should pay tax on this income. IRS says Ees should pay tax Held: Amts paid by Educo trusts are additional compensation to EE/TPs, so they should pay tax o According to this court, the EE is performing the services o Isnt the child performing the services? Analogy of Barry Bonds whose parents took him to all his practices as a child, etc. Do his parents owe money on the income he earns today? Here, child performs services, but of course parents dont owe tax on this income.

Lucas v. Earl, Poe, & Armatrout all deal w/ EARNED INCOME.

2. UNEARNED INCOME
TRANSFERS OF PROPERTY AND INCOME FROM PROPERTY Blair v. IRS (pg 681) A father has an interest in a trust as income beneficiary for life. The father assigned a portion of his right to receive income for life it his son. A trust is a contract where one party gives money to one party, the trustee, to manage it for another party, the beneficiary. Held: Assignee (son) is the taxpayer bc the assignment was a transfer of property the father gave up his control over the income This case if different bc there is a transfer of property If you have property, you gave give it away and the beneficiary pays the tax. If you give away your owned income, you still have to pay the tax. (Lucas v. Earl) Property and labor are different cant give away yourself, but can give away your property

40

Helvering v. Horst (pg 683) Father owns interest bearing bonds and gives interest coupons to son, but keeps the bonds. Held: Father is still the taxpayer bc he retained control over the source of the income But, if the father had given the son the entire bond (principal and interest), the son would pay the entire tax Today, there is no motivation to assign income to your child bc we have a Kiddie Tax (1(g)) the unearned income of a child under 14 is taxed at the parents rate.

Court establishes a line btw earned (taxpayer is earner) and unearned income (property based income - taxpayer is person who owns the property the donee).

SERVICES TRANSFORMED INTO PROPERTY Helvering v. Eubank (pg 690) Eubanks is an insurance agent and gets commissions, especially on the renewal commissions. He assigns the renewal commissions from past services. Issue: Did he assign income or property? o If income, then the assignor would pay the tax bc you cant make an anticipatory assignment of income o If property, then the assignee would be taxed Held: It was income bc no further services were provided o But, The court is divided Lucas v. Earl theory is the majority cant assign earned income Blair theory you can assign it bc its property based

HYPO: A is a programmer and develops a program, puts it on a disk and gives it to his son to sell to Microsoft. Who pays the tax on the income made from the program? 1(g) Congress accepts the distinction btw earned and unearned income. Bc for a child w/ earned and unearned income, there are different results. 1(g) at some level ratifies the case law. Congress has intervened in 2 ways Joint return so many of the issues are irrelevant now 1(g) child is taxed at the parents bracket

Helm v. Fitzpatrick (pg 691) TP invents something and gets a patent. He transferred the patent and the patent royalty rights to his wife and kids. Held: Assignees pay the tax bc this was a transfer of property.-- he gave away income producing property and any reversionary interest. (Blair case applies) Is this right? o Heims labor generated the royalties, so why is isnt this just like Lucas v. Earl> o He made the patent (like Lucas v. Earl) Main Point: Patent and patent royalties are considered property

ARTIFICIAL ENTITIES (trusts or estates, partnerships, corporations)

1. TRUSTS
Under certain circumstances, each of the 3 parties gets taxed (grantor, trust itself (beneficiary) & trustee). 671-678

GRANTOR TRUSTS

GRANTOR --- TRUSTEE --- BENEFICIARY Helvering v. Clifford His the grantor, beneficiary is his wife, H is also the trustee. At the end of the 5 year trust, the money goes back to the grantor (H). Held: Grantor is the taxpayer bc its not a valid trust Would the outcome be different if the trustee was a 3rd party? o Maybe it would be a better case for the wife to pay the tax

41

671 TRUST INCOME, DEDUCTIONS & CREDITS ATTRUBUTABLE TO GRANTORS AND OTHER OWNERS Clifford Principle Codifies Clifford When grantor is the trustee of the trust, grantor pays tax o Grantor has retained too much control/power, so they are still taxed even though state law recognizes the trust o In these situations sometimes the income is going to the beneficiary but the grantor pays the tax o Grantor is deemed to be the owner, even though he is not the recipient of the income If the following rules from 673-678 are trigged, then it is a grantor trust and the grantor is the taxpayer.

WAYS IN WHICH THE GRANTOR IS DEEMED TO BE THE OWNER (retains too much control) - 672 - 678

672 DEFINITION AND RULES Rules that say that if grantor (or person other then grantor) has too much control, grantor is considered owner and grantor is taxed on the income. 672-defintion and rules a. adverse party- any person having a substantial beneficial interest in the trust (he is adverse to the grantor b. Non-adverse party- person who does not have an interest in the trust. c. Related or subordinate party- person related to or subordinate to grantor d. any power given to granters spouse is deemed to be held by the grantor.

673: REVERSIONARY INTEREST 673(a) Clifford principle o If there is at least a 5% chance of the reversionary interest occurring, grantor is the owner

674 POWER TO CONTROL BENEFICIAL ENJOYMENT Grantor will be treated as the owner of a trust if he or a nonadverse party has control over the beneficial enjoyment of the corpus of income, without the consent of the adverse party.

675 ADMINITRATIVE POWERS If the grantor or grantors agent has administrative powers, then grantor is he owner and is taxed. If grantor has administrative powers (power to borrow, etc.) o Even if he gives the power to a non-adverse party (his lawyer, etc.) he is potentially taxable

676 POWER TO REVOKE If grantor has the power to revoke, he is taxable (even if he doesnt revoke)

677 INCOME FOR BENEFIT OF THE GRANTOR

678 PERSON OTHER THAN GRANTOR TREATED AS SUBSTANTIAL OWNER

NON-GRANTOR TRUSTS (ALL OTHER TRUSTS NOT COVERED UNDER 672 - 678) Starting w/ 641

These are bona fide trusts that are taxable to the trust (trustee pays) - If there is no distribution and the money is held, the trustee pays the tax One exception 653 - If there is a valid trust, then income distributed to the beneficiary is taxed to the beneficiary

641 IMPOSITION OF TAX Tax shall be imposed to the taxable income of estates or any kind of property held in trust, including: See Code for list of these things

643 DEFINIITIONS APPLICABLE TO SUBPARTS A,B, C AND D (a) Distributable Net Income: The taxable net income of the estate or trust computed with the following modifications (See Code)

42

651 &652 apply if trustee must distribute all the trust income annually: - tax beneficiary, deductible to trustee

651 DEDUCTION FOR TRUSTS DISTRIBUTING CURRENT INCOME ONLY Applies when trust is not accumulating any income nor distributing corpus a. if trust provides that all of its income is required to be distributed currently, (and doesnt provide that any amounts are to be paid, permanently set aside, or used for purposes of charitable donations), the trust can take as a deduction the amount of the income for the taxable year which is required to be distributed currently. b. Limitation on deduction if the amount of income require to distributed currently exceeds the distributable net income of the trust, the deduction shall be limited to the amount of distributable net income.- (only what it pays out) The trust gets a deduction for the amount of the income paid out to the beneficiary, so trust has no income, so does not pay tax A trust is taxed according to rates in 1(e)

652 INCLUSION OF AMOUNTS IN GROSS INCOME OF BENEFICIARES OF TRUST DISTRIBUTING CURRENT INCOME ONLY The income that is required to be distributed currently shall be included in the GI of the beneficiary

661 and 662 apply if the terms of the trust permits the trust to accumulate income (all trust income is not required to be distributed currently).

661 DEDUCTION FOR ESTATES AND TRUSTS ACCUMULATING INCOME OR DISTRIBUTING CORPUS a. deduction the trust gets a deduction for 2 things: 1) the amount of money that is required to be distributed currently. 2) Any other amounts properly paid or credited or required to be distributed for such taxable year.

662- INCLUSION OF AMOUNTS IN GROSS INCOME OF BENEFICIARIES OF ESTATES AND TRUSTS ACCUMULATING INCOME OR DISTRIBUTING CORPUS a. beneficiary must include in GI: 1) amounts required to be distributed currently, and 2) all other amounts properly paid, credited or required to be distributed to the beneficiary for the taxable year.

43

2. PARTNERSHIPS
701 PARTNERS, NOT PARTNERSHIP, SUBJECT TO TAX Partnership as an entity does not pay tax. Partners are liable personally for tax of the partnership. Partnership are flow through entities The Partner is the taxpayer, regardless of whether they get distributions of income

702 INCOME AND CREDITS OF THE PARTNER Each partners income tax is determined by his distributive share of the partnership. a. each partner, in determining his income tax, shall take into account his distributive share of the partnerships: 1) Gain/loss from sales or exchanges of short term capital assets 2) Gain/loss from sales or exchanges of long term capital assets 3) Gain/loss from sales or exchanges of trade/buz property 4) Charitable contributions 5) Dividends 6) Taxes 7) Other items of income, gain/loss, deductions, credits to extent provided by regulations prescribed by the secretary 8) Taxable income or loss, exclusive of items requiring separate computation. 702(c) GROSS INCOME OF A PARTNER: shall include his distributive share of the gross income of the partnership

703 PARTNERSHIP COMPUTATION

IRS v. Culbertson (FAMILY PARTNERSHIP) Father sells the partnership in a cattle ranch to his sons. IRS says the father is the taxpayer. IRS is denying that it is a partnership for tax purposes. Culbertson says children are taxpayers Held: Tax Court held father was the only taxpayer. Court of Appeals holds sons are partners and pay tax. Supreme Court says they dont know who the taxpayer should be so they remand to the Tax Court to reevaluate using different factors 701 rule is the partners are the tax payers in accordance w/ their shares TEST for partnership: In order for a person to be a partner, he has to perform services or invest capital o If the partnership is valid under Wyoming law, why shouldnt the children share in the tax? o A partnership is 2 or more people carrying on a business for profit 2 or more people as co-owners in business.

Today most entities formed as partnerships are formed as LLCs that have tax to corporation, not individual partners. This case led Congress to enact 704(e) 704 PARTNERS DISTRIBUTIVE SHARE The taxable income of the partnership is computed and then split among the partners See also 1366(e) If father gives property to son and son contributes $ or if father gives as gift and son participates, son is a partner 704(e) FAMILY PARTNERSHIPS o Congress modified Culbertson with this provision

Estate of Clarks Estate sued to recover interest portion of attorneys contingency fee paid in a personal injury action. IRS says TP pays tax on the entire amount and Lawyer pays tax on the 1/3 that he receives o This would leave the TP with very little Held: TP is not taxed for interest used to pay lawyers fees There is a circuit split on this issue and the Supreme Court has yet to hear any of these cases Is this right? o From Lucas v. Earl we know that you cant assign your income o Are Mr. Clark and his lawyer partners?

44

Brooke v. US (pg 709) (father gift to kidsleases propertykids pay income) TP deeds real estate to his children. Court appoints father as guardian of kids property. He leased the property to himself and used the lease money for expenses for the kids. Pays rent to the kids and can deduct his rental expense as a business expense and the kids pay taxes. Father argues he transferred the property to the kidsso it no longer belongs to him Held: Taxpayer wins KIDS are the TP

3. CORPORATIONS
Corporations are taxed as entities and there is a double tax regime o The corporation itself is the taxpayer o Dividends are income the shareholder under 61(a)(7) o There are no deductions to the corporation for paying out dividends, so this creates the double tax regime Corporate income tax is the same scheme as personal income tax

11 TAX IMPOSED ON CORPORATIONS Corporations are taxed for their taxable income 11(b) AMOUNT OF TAX

Foglesong v. IRS (pg 716) (Father forms corp. and gives dividends to kids) Foglesong formed his own corp. His kids owned a large portion of stock and the corp. paid dividends to them. Before he incorporates his salary is $120,000, after forming the corp. his corporate salary is just $70K and pays dividends to kids (he underpays himself to evade taxes) o The first $50,000 is only taxed at 50% o Here, the double tax is lower than the singe tax and he saves money Held: This arrangement works bc the court gives deference to the corporate form Today, under 1(g), this wont work anymore bc the kids are taxed at the fathers rate (unless they are students) 704(e)(2) To the extent that the father underpays himself, Congress wont allow it Father must pay himself a reasonable salary

Can 482 be interpreted to get the same result? Does 482 say the same thing as 704(e)(2)? Does 482 give the court authority to say that the corporate form is valid 482 ALLOCATION OF INCOME AND DEDUCTIONS AMONG TAXPAYERS Main pt: if a corp. controls 2 or more buzs, the IRS can allocate income btw or among them. If 2 or more orgs, trade/buz, (whether or not incorporated), are owned or controlled (directly or indirectly) by the same interest, the Secretary can allocate GI, deductions, credits or allowances btw the orgs/trades/buzs, if Secretary determines it is necessary in order to prevent evasion of taxes (or to clearly reflect income of org/trade/buz). Secretary has discretion but limited to whats reasonable.

In Fogelsong, the gov said that based on 482, it could allocate the income to Fogelsong.- ct said no. Post Fogelsong developments: Today, Fogelsongs arrangement would not work to reduce his taxes, bc we have kiddie tax, whereby a child is taxed on unearned income at parents rate. Also, the lower brackets for a corp. are not allowed for a personal service corp., as defined in 448(d)(2)

45

ACCOUNTING ISSUES OF TIMING


446 GENERAL RULE FOR METHODS OF ACCOUNTING (a) General Rule: Taxable income shall be computed under the method of accounting on the basis of which the taxpayer regularly computes his income (b) Exceptions: If no method of accounting has been regularly used by the tp, or if the method used doesnt clearly reflect income, taxable income shall be computed under such method that, in the opinion of the Secretary, clearly reflects income (c) PERMISSIBLE METHODS (1) Cash method pay tax when you receive it and deduct when you pay it (2) Accrual Method Pay tax when you earn it and deduct when you owe it (3) Any other method permitted (4) Any combination of the foregoing methods permitted 446(d) -if more than one business can use different accounting systems for each 446(f) -must get permission from Sec. to change and if not change not factored in

1. CASH METHOD
There are doctrines that have blurred the line btw cash method and accrual method: 1) constructive receipt and 2) Economic benefit 1) Constructive receipt for a TP using cash method, if he has an unqualified right to a sum of money and the power to obtain it, then he has constructively received it and it can be counted as current income even if he chooses not to receive it. However, there is no constructive receipt if the TPs control of its receipt is subject to substantial limitations or restrictions. 2) Economic benefit doctrine- a cash method taxpayer is considered to have received income as soon as the payor irrevocably sets aside funds for the TP in a manner that prevents the payors creditors from being

Amend v. IRS (pg 306) (sells wheat and tells guy pay me next year- not income this year) Tp uses cash method. He delivers wheat in 1944 but arranges to get paid in 1945. TP says that he should not be taxed until 1945, when he gets paid. Gov argues that he should be taxed in 1944, bc of constructive receipt doctrine. Gov argues that bc TP could have gotten paid in1944 (but chose not to), then the constructive receipt doctrine applies. Held: Constructive receipt doctrine doesnt apply tp has no right to the money now.

Pulsifer v. IRS (pg 311) ($ won in sweepstake--- not getting it till 1969- tax them now) A father (who does his tax returns on the cash method) and 3 kids won sweepstakes in 1964. but they dont get the $ till 1969. the $ is put into a trust for the children. In order to get the $ released, the father would have to file application for it. The gov wants to tax them now. Held: Tax them now bc of the economic benefit doctrine- they have an absolute right to the money- the money was irrevocably set-aside for them and they had an absolute right to it.

Thomas v. US (TP won lotto- not paid till next year but wants to be taxed now- NO!) TP on cash method won the lottery in 1992 but didnt get the money until 1993. in 1993, the tax rates went up. TP wants to be taxed as of 1992, so he can get lower tax rate- so he argues for the application of the economic benefit doctrine. (want to invoke Pulsifer). Held: Economic Benefit doctrine doesnt apply -- So he is taxed as of 1993. o ct distinguished Pulsifer, which stated that the sweepstakes winner is taxed in the year they won bc of the economic benefit doctrine. CT says that here in Thomas, the fund is not a segregated fund like it was in Pulsifer- so Thomas did not have an absolute right to the winnings in 1992.

46

DEFERRED INCOME Minor v. US (pg 324) (Dr. puts $ in a trust- no income now bc its accessible to creditors) TP is a doctor, employed by a corp. TP is also a SH and director of the corp. TP (Dr.) and corp. set up a retirement trust for TP. The corp. pays a salary and also puts $ into the trust for TPs retirement - this is an attempt to defer TPs income. Gov wants to tax him now on the $ going into the trust. Held: The trust is unfounded, can be forfeited, etc. so it is not taxable today. o Economic benefit doctrine does not apply- the funds were subject to the claims of the corps creditors there is risk of forfeiture. o We can argue that there was economic benefit and that it did apply bc the TP here is a trustee and a director of the corp.- he has control. Also he is choosing not to get the $ now.

Al-Hakin v. IRS (pg 330) ( agentpay me over 10 years- give me loan now---not taxed now) Agent was a TP (cash method) has k with player that agent will receive his fee over 10 years in equal installments. Agent and player enter agreement that player will now make him a loan equal to the total amount owed agent, and then each year, agent will cut off his fee from what he owes player.- so in effect agent is getting his entire fee now! Upfront!- gov wants agent to pay tax now on all assets of his fees. TP is the agent and get paid 5%, which is paid out over 10 years. TP wants the money early so he takes a loan from the player for the total amount and deducts the loan repayment for each month that he invoices the player. Held: TP only pays tax on the monthly portion of the loan -- agent DOES NOT have to pay taxes now at the time of the loan.- this was a loan, so no need to pay taxes on income upfront!- shocking case!

Problem pg 329 Cash basis taxpayer a. Taxed now on $600K? b. c. d. Least likely to be taxed now. Is this different than the guarantee from the insurance company? Does it matter which of the guarantors is safer?

Olmsted (pg 333) (agent retires-deals with insurance--pay him annuity--$ not taxable now) An insurance agent is retiring. He makes a deal with an insurance company- the company will give him an annuity ($500 a month for 15 years) in exchange for his future commission. The gov says that TP has too pay tax on all of the $ that he will be receiving upfront (now). Held: TP does NOT have to pay tax on all the money now -- Bc he does not have a right to all $ now (they wont give him all the money now if he asked for it).

2. ACCURAL METHOD
accrual method is used by most buzs. under accrual method, income is taxed when it is EARNED, and expense is deducted when they are OWED.- it is income when it is earned, regardless of when it is physically received. delay in the receipt of cash

All events test: income for deductions accrue when all facts/events occur that establish liability and can be established w/ accuracy Georgia School-Book Depository (pg 385) TP is on accrual method. TP is the middleman btw the state and the book company got paid by state from a state fund. He sent books to the state in 1938 and 1939 but, state fund didnt have enough money to pay him then. IRS wants to tax in 1938 and 1939. TP wants to be taxed later on. Held: TP loses, there is reasonable expectation that TP will get paid, so its income. Does all events mean all events or just all significant events? o If all significant events, then court is right bc hes done everything except file the paper work, so he should be taxed bc hes earned the income Court makes the argument that TP keeps delivering books so obviously he believes he will get paid.

47

PRE-PAID INCOME American Automobile Assoc. (pg 389) AAA getS prepaid for one-year membership. AAA is on an accrual basis, so they argue they havent earned the $ yet, so shouldnt pay taxes now until they earn the money Held: TP loses, must pay taxes on the $ collected now recognized when dues are received. o Accrual only matters if earned before paid, but if paid before earned, you are on the cash basis. What would happen if the membership is cancelled about paid? Did this matter the to S.Ct.? If its non-refundable, AAA has had an accession to wealth at time $ is paid, so they should pay taxes. When do they have tax paying capacity? Test under 446 is clearly reflect income If advising AAA, advise them to put the money away and not spend it until they have earned it.

455 PREPAID SUBSCRIPTION INCOME Prepaid income is included in GI for the taxable years during which the TPs liability to furnish a newspaper, magazine, or other periodical exists. (this did not help AAA bc it was regarding periodicals). Taxes is spread out during the time for which liability to perform exists This is essentially AAAs position This section was on the books at the time of the AAA case, so the court argues that bc Congress explicitly gave publishes this tax structure and didnt give it to other companies, they intended for other companies to pay tax on a cash basis. Does 455 apply to an online service to which you prepay? Is it newspaper, magazine or other periodical?

Congress responded to this case w/ 456. 456- PREPAID DUES INCOME OF CERTAIN MEMBERSHIP ORGANIZATIONS (e)(2) prepayments are NOT INCOME until services are performed. Under 456, AAA would have won. 456 permits membership orgs reporting on the accrual method to elect to spread prepaid dues over the period of responsibilities for the performance of services (but this period cant exceed 36 month) Prepaid dues income will be taxed in years in which the liability is earned AAA ultimately wins

Suppose you pay your gym membership dues one year in advance to a for-profit corporation. Under 456(e)(3), it would not apply to a profit making organization. This type of organization reports as money is paid. This section only applied to a not-for-profit corporation

DEDUCTIONS UNDER THE ACCRUAL METHOD


Deductions under the accrual method are subject to the all events test. Under the all events test, a future obligation is not deductible unless 1) the fact of liability is firmly established and 2) the amount of the liability can be determined with reasonable accuracy The all events test was codified in 461 (h)

General Dynamics v. US (ER deducts before claim is filed not allowed all events test not met) TP on accrual method set up a self-insured medical plan they reimburse Ees medical expenses. They set up reserves for services rendered. At the end of the year it claimed deductions for its obligations to pay for medical services that, that at the close of the taxable year, had been rendered but as to which the EE had not yet submitted claims to the company. Held: TP looses Cant take the deduction now bc expense has not yet been incurred. All events test has not yet been satisfied Test of deduction is liability. o Doesnt GD have the liability at the end of the year? Court relied on the all events test Dissent said tp could deduct now before claims were filed bc they read all events test as all significant events Question is how to interpret this test

48

461(h) CERTAIN LIABILITIES NOT INCURRED BEFORE ECONOMIC PERFORMANCE This provision is aimed at tax shelters specific to deductions for accrual basis taxpayer (4) Codification of All Events Test o A future obligation is not deductible unless (1) all events have occurred which determine the face of liability and (2) the amount of such liability can be determined with reasonable accuracy (1) In addition to liability being established before you can deduct and the amount being determinable with reasonable accuracy, this provision adds an element of economic performance test [ELEMENTS OF THE ALL EVENTS TEST] (2) This ECONOMIC PERFORMANCE TEST adds an element of the cash method o The result of the economic performance test is to mitigate manipulation of the accrual method. So, there are 3 ELEMENTS FOR DEDUCTIONS taken by accrual method taxpayers 1. A liability 2. That can be determined w/ reasonable accuracy and 3. There is economic performance, so it seems to be deductible in the current year.

HYPO: Call the local store at the end of the year and say you want to accrue expense by buying paper. If the paper is delivered on Jan 2, no deduction for the previous year. If you are aN accrual basis taxpayer, expense doesnt count, even if you send a check before the end of the year, until you are provided with the product. But, if you receive the paper before the end of the year but havent yet paid, there is (1) a liability (2) that can be determined w/ reasonable accuracy and (3) there is economic performance, so it seems to be deductible in the current year. In general, taxpayers resist the accrual method, but it is forced on them by the government. 448 requires corporations, partnerships or tax shelters to use the accrual method.

464 LIMITATIONS ON DEDUCTIONS FOR CERTAIN FARMING EXPENSES [Zelinskys Uncle Marvin & Borris the Bull story.] Aimed at tax shelter type behavior o Farming syndicate is bought prior to the end of the year. They accrue deductions before the end of the year and get a loss. Partners get to deduct the loss. Congress responded to this game w/ 464 o Now, when farming syndicate is bought at the end of the year, no deduction is allowed unless the is used or consumed. o This shut down the shelters that manipulated the accrual method 464(a) GENERAL RULE: For any farming syndicate, otherwise allocable deductions for food, seed, fertilizer, or other similar farm supplies shall only be deducted for the taxable year in which they were actually used or consumed

3. INSTALLMENT METHOD
453 INSTALLMENT METHOD Puts TP on cash basis- pro rata cash method of accounting The installment method allows sellers of property (not services) who are receiving payment on a deferred basis to pay tax on their gain only as they receive cash from the sale. Profit (income) is recognized ratably as payments are made to the TP. a. GENERAL RULE: Income from an installment sale shall be taken into account under the installment method. b. INSTALLMENT SALE DEFINED 1) Installment sale: means a disposition of property where at least 1 payment is to be received after the close of the taxable year in which the disposition occurs. Installment sales only pertain to property Must have a disposition of property where at least one payment comes in the following year [If you convey property in 2002 and get one installment in 2003, you are taxed in installments]

c.

DEFINITION OF INSTALLMENT METHOD: Under the installment method, the TP only recognizes a certain portion of income that year from the installment sale. To figure out the ratio of each payment amount that equals income figure out the ratio of gross profit (gain) to total contact price (what u r selling it for).

49

Example: TP sells property of $300 to be paid over several years. Gain will be $200, and total selling price is $300. The ratio of gain to total selling price is 200/300 = 2/3. S apply this ratio to each payment you receive, and the amount of the payment will be treated as gain (and this the amount that will be taxed) d. ELECTION OUT: TP has the option to elect out of the installment method (TP would do this if he is currently in a low bracket or knows that tax rates will go up)

f. (3) DEFINITION OF PAYMENT: for purposes of the installment method, payment may not be accomplished by a promissory note or other evidence of the indebtedness of the buyer. Suppose you sell a car for $12,000. [Doesnt apply if you are a car dealer or this is part of your inventory.] Total Contract Price $12,000 Adjusted Basis $8,000 Profit to be Realized $4,000 (Amt realized)

Gross Profit to Contract Price $4,000/$12,000 = 1/3 So, in this month, if you get a payment of $900, $300 is taxable. Under the installment method, it is not a cash method bc you get some $ tax free. The installment method puts us on a pro rata basis. Accrual method tp who makes an installment sale pays tax in this way. This is very favorable to the taxpayer

If you are on the cash method, you would pay tax on the entire $12,000 (if it is secured). ___________________________________________________________________________________________

4. INVENTORY ACCOUNTING
Hypo: Grocer has peas on the shelf. One way is to say that under 161 he takes a deduction for buying the peas. But this isnt a true reflection bc he really on exchanged cash for peas when he bought them, so he should only be able to take the deduction when he sells the peas. 471- GENERAL RULES FOR INVENTORIES (a) General Rule: Inventories shall be taken by such tp on such basis as the Secretary may prescribe as conforming as nearly as may be to the best accounting practice in the trade or business and as most clearly reflecting income

Today, these rules might be obsolete bc computers can record exactly which items were bought and sold. Adopt the accounting professions concept of taking inventories 3 step process o Count up inventory at the beginning of the year o Add on the cost of the goods that you bought during the year (# of cans bought during the year) o Take account on the last day of the year how many cans are left and take the difference btw this amount and what was left FORMULA for COGS = Opening inventory + Inventory purchased during the year Inventory left over Cost of goods sold during the year is the only deduction you can take If during the year, you buy cans at different prices, how to determine which cans were sold. o FIFO: Ones that were sold here the first ones bought o LIFO: Assume most recently purchased ones were sold

50

Problem #1 pg 582

Opening Inventory Acquired During the Year

End of the Year Left Amount Sold During the Year

100,000 100,000 200,000 100,000 100,000

(a) LIFO 100,000 * $3 = $300,000 deduction ($50K of taxable income for the year) LIFO produces a higher deduction when prices are going up. (b) FIFO 100,000 * $2 = $200,000 deduction (More income for the year;$150K of taxable income) Traditional accounting method is to use FIFO. Thor Power Tool Company v. IRS TP has excess inventory that they hold on to to satisfy their customers that dont want to buy the new models. Accountants tell TP that they have to write-down this inventory to be truthful about the inventory. Write-down means to show the price of the items as the lower price they are worth today (so to compare to the widget example above, instead of showing each widget as $2/widget, must show as $0.50/widget). This write-down helps TP for tax purposes bc they are taking a deduction. Court doesnt let them take a deduction for this write-down. o 2 things must be shown to take a write-down as a deduction You will offer it at lower cost or Scrap it o Neither event occurred bc they were still selling the inventory at the higher price The best accounting method would be to write-down the inventory, but the TP is still selling the inventory for the higher price. What happens if you cant sell the peas bought for $2 for less than 50 cents? If you show something is worth $2 bc thats what you paid, but now its not worth that much its not a true picture to the public. In a situation where you can demonstrate that current market price is lower than the cost, you utilize the lower cost. Main Point: Cant deduct inventory until you get rid of it COGS. ___________________________________________________________________________________________

5.ORIGINAL ISSUE DISCOUNT (OID)


Puts everyone on the accrual method Can lend money and not take periodic interest payments and just get one lump sum.

Suppose you borrow $600,000 and agree to repay $1m in 5 years. Cash method: $400K of interest in year 5. TP would pay the $400K in taxes in year 5. The OID is the $400K this is the unstated interest Instead of the interest being paid back in increments, it is paid in 1 lump sum at the end of 5 years. But, this a mount is treated as interest earned (for the lender) in increments over the 5 years. But, Congress wants to put everyone in on the accrual method in this situation. o Lender is reporting the interest as income (before they even receive the interest) o Borrower is deducting daily for the interest payments (this is a windfall for the borrower bc gets deductions early) 1273(a)(1): DETERMINATION OF AMOUNT OF ORIGINAL ISSUE DISCOUNT OID is the excess (if any) of o (A) The Stated redemption price (the amount to be paid back ($1M in the hypo), over o (B) The Issue Price (the amount lender is loaning ($600K in the hypo) Basically OID id the amount of money I will end up paying someone for letting me borrow money. Its not really interest bc I just pay them the money at the end when I return what I borrowed- but Code wants me the borrower to deduct such an expense over the amount of years b4 I pay it back and wants the lender to the money he is making from this lending now as GI over the amount of years till he gets paid. FORMULA: Stated Redemption Price Issue Price = OID $1,000,000 -$600,000 = $400,000

51

1272(a)(1) CURRENT INCLUSION IN INCOME OF OID GI FOR LENDER OID is included in the lenders income on a daily basis over the next 5 years even though he doesnt actually get the interest until year 5 (based on hypo) Puts everyone on the accrual method Interest accrues on a daily basis even though there has been no cash Pay tax on the interest based on this daily interest that is accruing

163(e)(1) ORIGINAL ISSUE DISCOUNT DEDUCTIONS FOR BORROWER Allows the borrower to take a deduction for interest owed that hasnt yet been paid (must be trade or business, cant be personal ,etc.) Same amount that the lender is putting into income on the theory it is being earned even though not yet paid, the borrower gets to deduct. ___________________________________________________________________________________________

6. ANNUAL ACCOUNTING
We do accounting of taxes based on the calendar year- sometimes this leads to perverse (mean/bad) results. There are times when events and transactions that take place in 1 year may have an impact in a subsequent year: Burnet v. Sanford & Brooks (pg 155) (pre-172) TP has a K with the govnt. Tp lost $ on the K over 4 years (In 1913 they lose $. In 1914, they have gain. 1915 they have losses again) and deducted losses. In 1920, TP gets a judgment for $176,000 dollars to make up for the losses. The IRS argues TP owes $ now in 1920. TP got this $ w/o any deductions, so they owe money. TP says this payment is not income, bc they didnt make a profit they simply broke even. Held: IRS wins Payment is INCOME and therefore taxable o All receipts are part of GI regardless of whether or not there is profit So even if its a repayment for past expenses, it is still taxable

Congress responded w/ 172 Under this provision TP in Burnet could have carried the net operating loss forward to 1920 and taken the deductions.

172 NET OPERATING LOSS DEDUCTION Congress has modified the harsher results that occur w/ annual accounting period

Net operating loss (NOL) deduction exists when the deductions available to a business exceed tps income. NOL = excess of deductions allowed -- GI (a) Deduction allowed for an amount equal to the aggregate of: o (1) the net operating loss carryovers to such year, plus o (2) the net operating loss carrybacks to such year (b)(1)(A) Can open up the prior 2 years and get a refund (carryback) or can use the deduction for the next 20 years (carryforward) (b)(3) TP can waive the carryback (d)(4) Only trade or business losses are carried back and forth, but individuals cant do this o So today, the Sanford case wouldnt even be an issue ___________________________________________________________________________________________

7. CLAIM OF RIGHT
North American Oil v. Burnet (pg 161) (States the Claim of Right Doctrine) TP and gov were fighting over property.. In 1917, a ct ruled that the property belongs to the TP. But, gov appealed and in 1920, ct held the property was the tps and the case was over. IRS wants tp to pay taxes on income for the property as of 1917 says tp had a claim of right to the property as of then. TP wants to pay in 1916 and 1922 (Tax increased bc of WWI after 1916) Held: The tax is due in 1917 bc tp had a claim of right to the property then Courts test to determine when to get taxed o When you have a claim of right w/o restriction as to its disposition

52

US v. Lewis (pg 165) (pre-1341) Tp is a cash basis taxpayer who erroneously received a bonus in 1944. In 1946, tp had to pay back a portion of the bonus. Tp wants to reopen 1944 and deduct (rates were high); IRS wants to deduct in 1946 (rates were low) Issue: When should Tp have to take the $11,000 deduction? Held: TP loses and he has to deduct in 1946 rather than recalculate for 1944 o Seems right bc he was taxed when he had the cash and now doesnt have the cash, so take the deduction now.

Congress responded to Lewis w/ 1341. Today, under 1341, the tp would have a choice either take deduction in 1946 for the loss, or reopen 1944 and get a credit for the amount he shouldnt have paid. Congresss solution to Lewis is that TP gets the lesser of 2 options Get whatever is better for the taxpayer

1341 COMPUTATION OF TAX WHERE TAXPAYER RESTORES SUBSTANTIAL AMOUNT HELD UNDER CLAIM OF RIGHT RULE: If an item was included in gross income for a prior taxable period bc the tp appeared to have an unrestricted right to such item but now doesnt have a right to the item, a deduction is allowable for the current taxable year as long as such deduction exceeds $3,000 tp has a choice to: o Take the loss deduction in the current year, or o Claim a credit in the current year for the tax that would have been saved by not paying tax on the item in year 1 (this is like reopening the old year)

53

CAPITAL GAINS & LOSSES


View as a modification of the basic structure an exception If you are in the category of a long-term capital gain, you can tax at very favorable rates A Capital Asset is a certain type of property (NOT SERVICES). When a capital asset is sold, thus there is a capital gain, the gain is taxed favorable. When there is a loss, the loss is not treated favorably. TP wants capital gain and ordinary loss (not capital losses) When the issue is capital gain, the tp wants it to be a capital asset, but when its capital losses, the IRS wants it to be a capital asset.

1221 CAPITAL ASSET DEFINED Defined by what is not a capital asset (a) Capital Asset means property held by the tp (whether or not connected to trade or business) o Capital Assets DO NOT include: (8 exceptions) INVENTORY EXCEPTION: Stock in trade or business of the tp or other property of kind which would be included in inventory of tp if on hand at close of business year, or property held by the taxpayer primarily for sale to customers in the ordinary course of business DEPRECIABLE PROPERTY FOR TRADE/BUSINESS: Property used in trade or bus. that is subject to depreciation, or real property used in business COPYRIGHT, LITERARY, MUSICAL OR ARTISTIC COMPOSITION, OR A LETTER: Held by the tp who created it or whom it was created or whose basis is determined by reference to that creator ACCOUNTS OR NOTES RECEIVABLE: Acquired in the ordinary course of business for services rendered or from sale of property A PUBLICATION OF THE US GOVERNMENT: COMMODITIES DERIVATIVE FINANCIAL INSTRUMENT: Held by a commodities derivatives dealer HEDGING TRANSACTIONS: Which is clearly identified as such before the close of the day on which it was acquired SUPPLIES: Of a type used or consumed by the tp in the ordinary course of business of the tp

Be Careful of Difference btw Capital Expenditure and Asset Capital expenditure (263) The issue is whether its deductible or not Capital Asset The issue is whether the tp is entitled to favorable treatment when he sells something 1222 OTHER ITEMS RELATING TO CAPITAL GAINS & LOSSES Difference btw short-term and long-term capital gains/losses Only long-term capital gains get the favorable treatment short-term gets taxed at the ordinary income rates Under 1(h) capital gain is taxed for lightly than regular gain (income) 1) Short term capital gain gain from sale/exchange of capital asset held for not more than 1 year 2) Short term capital loss loss from sale/exchange of capital asset held for not more than 1 year 3) Long term capital gain gain from sale or exchange of capital asset held for more than 1 year. 4) Long term capital loss loss from sale or exchange of capital asset held for more than 1 year.

1211(b) LIMITATION ON CAPITAL LOSSES (NON-CORPORATE TAXPAYERS) Capital Losses are limited, which is the flip side of capital gains Individuals can only deduct $3,000 of capital losses ($1,500 for married indiv. filing separately) Over that amount, can only deduct capital losses to the extent of capital gains (I can only deduct $3,000 of capital losses, but if I have more to deduct, I can only do so up to the same amount as I have capital gains) This limit applies to both short-term and long-term capital losses

1212(b) CAPITAL LOSS CARRYBACKS AND CARRYOVERS If capital losses exceed capital gains, you can carry them over to the next year, indefinitely until they are used up

54

1(h) RATES FOR CAPITAL GAINS See problems on handout Most capital gains have a maximum tax of 20%, while most other income has rates that go much higher. Short-term gain (held under 1 year): Taxed at like ordinary income Long-term Capital Gain (held more than 1 year): Taxed favorably at 20% or 10% Collectibles and 1202 Gain: Taxed at 28% Real Estate related gain (1250 capital gain): Taxed at 25% 5-year capital gain (took effect in 2001)

See handout!!! WHAT QUALIFIES AS CAPITAL ASSETS? Van Suetendael v. IRS (pg 752) (securities not for sale to customers) TP has losses from buying and selling stocks and bonds. So, he wants them to be ordinary losses. IRS wants them to be capital loss. Held: TP looses, securities were capital losses o Under 1221(a)(1), the court says these were not securities held primarily for the sale of customers in the ordinary course of trade or business

Biedenharn Realty v. US (pg 756) (property not capital asset bc held primarily for sale to customers) TP originally buy the lot for farmland and for investment. Then, later on they break it up to sell as residential property so they make improvements to make it residential. Issue: Does this activity turn what was an investment into inventory, and thus no longer a capital asset? Held: Gain is ordinary gain. Not a capital asset bc it was held for sale to customers. -- Falls under 1221(1) exception. If it was investment property, its capital assets

Selling to a developer is a capital asset. But, if you develop on your own to sell, its ordinary asset. Corn Products Refining Co. v. IRS (pg 769) (futures are not capital assets bc its like inventory) TP, a manufacturer of corn products, bough corn futures to protect its supply of corn. TP sold some of the futures for profit. TP claims the futures were capital assets (bc they had gain). IRS says its ordinary gain. Held: TP looses Corn futures are an ordinary asset (not a capital asset) Falls under 1221(1) exception Analysis under 1221(1) o This K is property o Held by the TP o Stock in trade No, bc tp isnt in the business of selling corn futures, this is just an input to their business of selling corn products o Sale to customers in ordinary course of trade or business No, corn futures is not their ordinary business The court got this wrong it doesnt fall under the exception to capital assets under 1221(1)

If its property that is not inventory, it is capital asset (like an insurance contract).

Arkansas Best Corporations v. IRS (pg 773) (stocks are capital assets) TP is a holding company that purchased shares of a bank. The bank went into foreclosure and tp has a loss upon selling the stock. TP claims the stock was not a capital asset (it purchased the stock as a business inventory) so its an ordinary loss. Under Corn Products, TP has a good case for ordinary loss bc its their ordinary activity Held: But, TP loses ct says loss arising from sale of the stock is a CAPITAL LOSS Court narrows Corn Products to hold just that hedging transactions are an integral part of a business inventorypurchase system, so its an ordinary loss. This case is inconsistent w/ Corn Products. Does this really fall under the exception in 1221(1)? o They have no inventory bc they dont sell to customers.

Congress responded w/ 1221(a)(7) &(8).

55

Hort v. IRS (Pg 780) (payment for cancellations of a lease is like rent ordinary income) TP (lessor) receives a lump sum payment when lessee terminates a lease. TP says gain (amt received from tenant) is capital gain. Held: TP loses Payment is ordinary income (not capital) - 61 o Similar to if a stockholder chooses btw collecting dividends or selling stock selling stock is usually capital gains. o If selling stock is capital gain, isnt selling the lease back like this?

McAllister v. IRS (pg 787) (a life interest like a trust is a capital asset) Father leaves a trust in his will giving income to his daughter-in-law (TP) for like with remainder to his son. TP has debts, so she sold her interest for a lump sum of $55,000 (this was much less than what the income was worth). TP says she has a capital loss. IRS says its ordinary income, so taxable. Held: Life estate was a capital asset, so payment received to terminate the trust is a capital loss. Dissent says that this money was a substitute for future payments, which would be taxable as ordinary income, so the consideration is ordinary income.

Are Hort and McAllister reconcilable?

IRS v. PG Lake (pg 792) (oil repayments are not property rights so not a capital asset) PG was a lessee it had a right to drill on owners land. PG owed $ to its president. In order to repay president, PG assigned him oil payment rights. PG says that the oil payment rights are pripoerty, and that this was a sale of a capital asset, so PG has capital gain (gain is the discharge of indebtedness) o Owner of the parcel where the gas is (farmer) gives rights to the driller (developer) this right is an Oil & Gas Lease. Developer then has a Working Interest. Lessee pays lessor a percentage of the amount of oil he drills. But, developer has no money and sells an Oil Payment Right to investors. This Oil Payment Right is capped. Lake has a working interest and gives an Oil Payment Right to their president. PG Lake rips up the promissory note to its president and gives him the Oil Payment.The company reported this transaction as a capital gain. The IRS says the purchase price was taxable as ordinary income. They all agree its discharge of indebtedness. But, this is not accurate bc he is still owed something He is still owed payment on the Oil Payment (before it was called debt). Held: Consideration recd for oil payments is ordinary income Oil payments rights are not property, so not a capital asset.

Problems pg 795 1. (c) TP owns right to take half of the water flowing along a river and sells this right for the next 10 years for a lump sum payment. Is this a capital asset? Is he like the landlord who gets the water back. If he sold it in its entirety, it would be ordinary income. Under PG Lake? (d) TP pays a lump sum for the right to use as a lessee certain business premises for 10 years and then subleases those premises for 6 years for a lump sum in advance. What is the impact on the owner of the premises? Ordinary income bc it looks like rent For lessee: $60,000 is basis bc thats what he paid for it When the lessee sells the lease (sublease): He paid $36,000 for 6 years, but makes $40,000 for the 6 years. What is the profit of $4,000 capital or ordinary? Does it matter if his business is subleasing or is this inventory?

Miller v. IRS (pg 819) (movie rights are a personal right not a property right NOT CAPITAL ASSET) Tp sells rights to her husbands name. She gets $400K for the rights. T claims this is a capital gain. Held: This is not property, so not a capital asset Its an ORDINARY ASSET o Her interest in the rights have not yet been recognized legally the rights are not property they are personal rights, so not capital o Today, this is property If its a capital asset, what is her Basis o 1012 what did she pay? - $0 o Did she inherit the right from him? If yes, then the basis is the FMV, so no gain If she didnt inherit it and its intrinsic to her, then she has no basis.

Requirement of a Sale or Exchange to be a Capital Asset Under 1222, is it capital only if there is a sale or exchange?

56

Helvering v. Hammel (pg 835) (a capital loss is a loss from the sale forced sale = sale) Property is disposed of by foreclosure and TP lost $. TP claims his loss is an ordinary loss, not a capital loss argues a capital loss must only be from sale of a capital asset (1222(2)) Held: TP loses It is a sale so a capital loss o If there was loss from being stolen, it would be an ordinary loss under 165

This is the law to date since Congress has not changed the statute. 1231 PROPERTY USED IN THE TRADE OR BUSINESS AND INVOLUNTARY CONVERSTION This is the best category for property to be in gain are treated as capital gains, and losses are treated as ordinary losses To qualify for 1231 treatment, the tp must either sell or exchange property: o Which is held for more than a year and is used in the tps business or o Suffer an involuntary conversion with respect to certain bus. and/or capital gains property that is held for a more than 1 year (long-term) (b) The property that qualifies as 1231 property means property used in the business of a character which is depreciable (equipment and real estate), and is held for more than 1 year (a) If there is a net 1231 gain, the gain is treated as capital. If there is a net 1231 loss, the loss is treated as ordinary.

1202 PARTIAL EXCLUSION FOR GAIN FROM CERTAIN SMALL BUSINESS STOCK Qualified small business stock held for more than 5 years. Gives a favorable tax treatment for stock ownership Not a traditional capital asset provision

1001(e) CERTAIN TERM INTERESTS General Rule: In determining gain/loss from the sale or other disposition of a term interest (life estate), that portion of the adjusted basis of such interest (determined under 1014 or 1015) shall be disregarded

165(g) WORTHLESS SECURITIES General Rule: If any securities, which is a capital asset, becomes worthless during the taxable year, the loss resulting from it shall be treated as loss from the sale or exchange of a capital asset. Security Definition: o Share of stock in a corporation o Right to subscribe for or receive a share of stock in a corporation (stock options)

Bond, note or certificate or other evidence of indebtedness issued by a corporation or by a government w/ interest coupons in registered form

57

TAX SHELTERS
Tax Shelter: An investment or transaction structured to achieve tax avoidance A tax shelter is nothing more than the packaging and use of tax provisions (deductions) that weve already seen. Tax shelter is people using provisions of the Code. To determine if its an abusive tax shelter: Look for when a tp has a positive cash flow, but he has a tax loss from depreciation deduction CLASSIC TAX SHELTER REAL ESTATE TAX SHELTER HYPO: M borrows from the bank to buy an apartment building. He gets $5,000 of rent. Ordinary income under 61 Pays tax of $1,000 Deductible under 164 Pays utilities, repairs, of $1,000 Deductible under 162 & 212 Pays mortgage interest $3,000 Deductible under 163 Seems like a net of zero But, there is depreciation of $1,000 Deduction 168 Seems like a loss of $1,000, but its a paper loss bc he hasnt paid anything for the depreciation. This loss is deductible under 165.

Why do this? -- M has a big salary, so with the deduction, he taxable income goes down. He hasnt really suffered a loss. If he is taking the deduction for depreciation while the value of his property is actually going up, he isnt suffering a loss its just a paper loss. Take losses w/o there being any outlay. The Congress has said that under some circumstances, this result produces an unacceptable result a tax shelter. Even though legitimate deductions are being taken. NONRECOURSE BORROWING Borrowing w/o being liable. 3 ways to borrow o Borrow with NO obligation to repay. o Borrow w/ personal guarantee of paying back Recourse o Nonrecourse you can take the collateral, but cant come after the borrower The entire tax shelter phenomenon turns on NonRecourse lending There is no interest deduction bc 163(a) says deductions for interest paid on indebtedness. If you dont treat that debt as real, there is no basis bc basis is cost.

Crane v. IRS (pg 201) TP inherits the building from her husband. Parties agree FMV is $262,000. The amount of the mortgage is $262,000. So, she has no equity bc the building is worth the same amount as owed (shady!). This is nonrecourse debt if the bank forecloses, TP doesnt have to make up the difference. The bank lent to the husband and forgot to get Mrs. Crane to guarantee it. o If the value of the building goes up, sell or pay the mortgage o If the value goes down, stick it to the bank She is in a very strong position She takes depreciation deduction of $25,000. In 1938, the bank forecloses so she sells to a 3rd party and she gets $2,500. TP claims her basis was zero so she realized $2,500 Court says her basis (adjusted Stepped up basis FMV minus depreciation) is $262,000(inherited basis 1014) minus $25,000 depreciation (1016) = $237,000. So she is basically relieved from the entire debt, and that is the amount realized. Does it make sense to say she is relieved from debt when she was not liable bc it was nonrecourse? This was a tax shelter for TP thats why she held it. Held: TP loses o If she deducted the cost, she shouldnt be allowed to use it again, so an adjusted basis makes sense o Declaring nonrecourse debt to be real debt was a victory for the IRS

This makes sense bc she lowered her taxes for many years and now has to pay taxes.

58

Tufts (pg 211) TP bought building w/ $40K of his own money and $1.8M n-recourse loan. TP deducted $400K in depreciation. TP used this deduction to offset other income. Then tp sold the building for a small sum (buyer took on the debt). Held: TPs amount realized includes the amount of the loan o TP has recognized gain in the amount of his previous depreciation deduction, so tp had t pay taxes on a gain of $400,000 $1.8M (amount realized) -- $1.4M(adjusted basis) = $400,000 (gain) o Makes sense bc deal lost money in the real world but made money in the tax world bc they took deductions Stands for the proposition Crane decision nonrecourse loans are real loans generated a lot of bad tax shelters. Bank could have taken the keys back and showed a loss of $400K to fix this problem Main Point: Nonrecourse debt is really debt -- Interest is deductible

After Crane and Tuft, there was possibility of tax shelter- TP could buy a building with a loan, depreciate, use the depreciation deductions to offset other income, and then upon selling, pay tax on the amount previously depreciated. This accomplishes tax deferral. Cottage Savings Assoc. (pg 261)** FACTS: TP is a S&L assoc. that does many of these garbage mortgages. If 2 banks exchange mortgages, they dont have to tell their shareholders about the loss. But, for tax purposes, they will show a loss to be able to get a deduction. Issue: Is this a sale or disposition under 1001? (Is it an exchange if so, there can be a loss) Holding: S.Ct. says yes and allows the deductions o Says there is a material difference in the loans the banks swap Under 446(a), the court could have said that they wont allow unacceptable accounting methods Why did they allow the deductions? They are allowing shareholders to think they have a good investment, when they really dont. If TPs take write-down, must tell shareholders, if dont want to tell shareholders, cant get the deductions They way they did do it is a fraud. o The Court was wrong and inconsistent w/ the Code (446(a)) 172 Carry Back Losses

ANTI-TAX SHELTER LEGISLATION


Tax shelters are classic Code provisions w/ nonrecourse debt. Classic tax shelter is something Tp uses to borrow money gives high basis, generates a loss, and losses are deductible (1065) Congress has adopted a variety of provisions aimed at tax shelter behavior -- Provisions in the Code designed to stop tax shelters.

1. BASIC DEDUCTIONS
163(d) LIMITATION ON INVESTMENT INTEREST Cant deduct investment interest, except to the extent you have investment interest HYPO: M buys land w/ nonrecourse loan. Takes interest deductions shows a loss bc no income, but he pays mortgage. This section prevents tax shelter w/o it tp could take deduction for interest w/o having income

263A CAPITALIZATION AND INCLUSION IN INVENTORY COSTS OF CERTAIN EXPENSES Requires a lot of things to be capitalized that were previously deductible Inventory costs are capitalized TPs have a strong incentive to deduct rather than capitalize This section is designed to create a matching Take deductions as you report income

183 ACTIVITIES NOT ENGAGED IN FOR PROFIT No deductions for hobbies Prevents certain behavior

59

2. NONRECOURSE DEBT SHELTERS


465 DEDUCTIONS LIMITED TO AMOUNT AT RISK Limited attack on nonrecourse debt For a borrowed amt to be considered at risk the TP must be personally liable or put property as security for collateral If not personally liable, the loss is not allowed It is carried over under 465(a)(2) Trigger is that tp is not at risk Losses not currently deductible may be carried over to the next year 465 significantly reduced the activity individuals could engage in as a tax shelter Today, this section applies to ALL ACTIVITIES Only affects tps borrowing on a nonrecourse basis

E.g. M puts in $1 to buy building and borrows $9 (nonrecourse). TAX SHELTER Basis = $10 Deduct interest, utilities, taxes, etc. (162, 164, 168, etc.) o (Loss of $1.50) Today, M can only deduct up to $1 bc thats the only amount he is at risk for. (465) Carryover of $0.50 (465(a)(2) In year 1, he can put in another $1.50 to take the deduction. This probably wont help. He would put in the additional money when it makes sense economically if the building goes up in value and you want to own more of the building] In year 2, if there is loss of $1.50 again, he cant take deduction bc $1 was already used in year 1, so carryover $1.50. To get around this, M must guarantee $9 loan. Under 465, it forces people to make economic decisions What was wrong in Tufts was that they were making crummy deals bc it didnt matter

In year 2, he sells for $13 (amt. realized) o $10 basis o $3 Gain o Then he can use the losses that have been carried over, bc now he has income o $3 - $2 (carryover losses): Pays tax on $1

465 only prevents a loss deduction Can still deduct if income is generated take deduction against the activity. If in year 3, he now has a total of $3.50 in carryover deductions and has gain of $3: $3 -$3.50 ($.50) Cant take a deduction for this loss

469 PASSIVE ACTIVITY LOSSES Trigger is tp is passive (not active) Puts another layer on tps ability to deduct Prevents being in business of purchasing losses 469(c): Defines passive activity So, even if you sign at the mortgage and are at risk and satisfy 465, if you dont participate, you cant use the losses against your salary income 469(d): Defines passive loss o Also only lets use the passive loss against other passive income, but cant use it against other active income 469(g) But, once he sells tp can use the losses against his salary Seems that 183 and 469 overlaps Congress rarely repeals provisions as they add, so there is overlap in many examples

Knetsch (pg 624) Borrows against the annuity 163(d) is this really indebtedness? Holding: TP isnt allowed to take interest deduction

60

3. ALTERNATIVE MINIMUM TAX (AMT)


Ultimate penalty on tax shelters It prevents tp from paying no tax bc of deductions o Aimed at high income tps with a lot of deductions o Requires calculating taxes twice and pay AMT if its higher than regular amount o Slower depreciation scales

55 ALTERNATIVE MINIMUM TAX IMPOSED AMT take taxable income, add to it the deduction, and impose a lower tax rate. But the resulting tax is payable only to the extent that it exceeds the normal tax. 1) Take taxable income, add back in deductions = AMT new taxable income 2) If AMT new taxable income is higher then regular taxable income, then take the AMT new taxable income minus regular taxable income = difference 3) Take difference minus $45000 = taxed at AMT rate (26% or 28%) 4) TPs final tax = regular taxable income plus (difference 45,000 taxed at AMT rate) Owe the excess of minimum tax over the regular tax If min. tax is higher owe min tax, if lower than regular owe regular To trigger AMT must have income of at least $45,000 That figure meant a lot more when the provision was enacted 55(d) Sullivans Article: AMT will start to affect regular filing tax payers rather than just high income people as it was intended

56 ADJUSTMENTS IN COMPUTING AMT TAXABLE INCOME 56(b) AMT for Indiv. o No deduction for income taxes o Why dont we just abolish 164? Medical Expense o Normally, deductible if exceeds 7.5% o 56(b)(1)(B) 10% rule Current projections are that more and more people will trigger AMT o Bc numbers of AMT are not inflation adjusted

4. QUALIFIED PENSION PROFIT SHARING


401, 408, 219 Tax benefit pensions provide a deduction under 404 employer gets deductions for contributions to employees pension plan. - The pension plan grows tax free 401 regulation of pension plans give requirement for a plan in order to qualify for tax benefits.

61

5. STOCK OPTIONS
421(a), 422, 83 LoBue v. IRS (pg 344) 3 Events: o 1 - Options are given to him: Option is issued LoBue on first day of employment for price A. o 2 - Exercises his option: He chooses to exercise his option at some time down the road. He paid A for the shares and when he purchases, FMV is B. o 3 - Sells stock: He will sell the stock at price C. When to tax? When is the realization? o Event 1: Dissent says tax when he gets the option. Is an option like a lottery ticket? o Event 2: Difference btw A and B Tp argues that he doesnt have anything at this point bc he hasnt gotten FMV yet. o Event 3: Difference btw C and A is income (gain) (1012 basis is cost) If he dies, tp is a big winner bc his basis moves to C (1014) Held: Tax at event 2 (exercise of the option) If price goes below A and courts rule of taxing at event 2 o he has a capital loss Same result if tp is taxed at event 3 and price goes below A capital loss But under 1001, no recognition until sale or disposition Event 3

Over time, Congress has adopted all three of these rules. 83 PROPERTY TRANSFERRED IN CONNECTION WITH PERFORMANCE OF SERVICES Codifies LoBue test tax at difference btw A and B Lucas v. Earl theme tax whomever performs services 83(e) o If option isnt tradeable, this rule doesnt apply o So, today if you give an EE a stock option you must say the option is not transferable If an option is given that can be turned into cash, it is taxed at Event 1 when option is granted If its a tradeable option, taxed at Event 1, if not, taxed at event 2

421 (a) STOCK OPTION RULES Tax at Event 3 when stock is sold Best alternative bc you defer tax and if you die before you sell, you get step up basis

422 INCENTIVE STOCK OPTIONS Rules tp must meet to get taxed at event 3 (when stock is sold)

Like 1341 you pick whats best to pay tax. Maybe youd rather be taxed at Event 1 if the value of the option is low then can avoid tax at Event 2 when you exercise the option especially if FMV really goes up. Event 1 get taxed if transferable Event 2 taxed normally if non-transferable (83) Event 3 taxed if meet 422 test 421(a) Choice is to tax at Event 1 or 2, and 3

62

6. DEPRECIATION DEDUCTIONS
1245 and 1250 Depreciating a property causes deferral of income and when TP does get the income upon sale, its capital gain. 1245 and 1250 say that part of that income will be ordinary when TP sells property that he ahs taken deprecation deductions on, the gain is ordinary to the extent that TP took depreciation deductions.

1245- GAIN FROM DISPOSITION OF CERTAIN DEPRECIABLE PROPERTY In the case of personal property such as tractors and other equipment and machinery, any gain upon sale is treated as ordinary income to the extent of prior deductions for depreciations. Example: TP buys tractor for $10,000, and he deduct deprecation for $6000, leaving adjusted basis of $4000. He sells for $11,000 (amount realized). He has a gain of $7000 - $6000 of that gain is treated as ordinary income, not capital gain. (Bc $6000 was depreciated) and $1000 is treated as capital gain.

63

You might also like